uWORLD Pulmonary

Pataasin ang iyong marka sa homework at exams ngayon gamit ang Quizwiz!

______________________ are yeast that form peudohyphae.

Candida > Candida infection is usually not associated with pulmonary infiltrates or lymphadenopathy

________________ in the cell wall of staphylococcus aureus helps prevent opsonization by binding the Fc region of immunoglobulins. S aureus is a common cause of tracheitis, which has a clinical presentation (eg, stridor, fever, respiratory distress) similar to epiglottitis

Protein A

What parameters account for the difference between *minute ventilation* and *alveolar ventilation*

Dead space volume

_______________________ causes pulmonary disease in immunocompromised patients. This fungus is seen in tissue sections as *septate hyphae* with V-shaped branching.

Aspergillus fumigatus

uWORLD: A 36 yo woman comes to the physician due to dyspnea and weakness. She has no other medical problems. her moth had similar symptoms and died at age 42. After an extensive work-up, lung biopsy is performed. Light microscopy of the tissue sample shows medical hypertrophy, intimal fibrosis, and decreased intraluminal diameter of the small branches of the pulmonary artery. The patient is subsequently scheduled for lung transplantation. Which of the following medications is indicated for management of her condition during the waiting period?

Bosentan > Bosentan is a competitive antagonist of endothelia receptors used for treatment of idiopathic pulmonary arterial hypertension > *Pulmonary hypertension* causes specific morphologic findings in the branches of the pulmonary artery, including increased arteriolar smooth muscle thickness (*medial hypertrophy*), *intimal fibrosis*, and significant *luminal narrowing*. In the setting of severe hypertension, lesions can progress to form interlacing tufts of small vascular channels called *plexiform lesions*. > These changes can occur in both pulmonary hypertension due to underlying lung, vascular, or cardiac disease and in idiopathic or familial pulmonary arterial hypertension (PAH).

uWORLD: A 26 yo medical student is exposed to a patient who suffers from active pulmonary tuberculosis infection. Two years later, the asmtomatic medical student presents with a chest radiograph finding of a small calf iced lesion in the right lung field. What cell contributed most to the containment of the infection?

CD4+ T lymphocytes and macrophages > Pulmonary tuberculosis infection is controlled through the action of CD4+ TH1 lymphocytes and macrophages. These cells work together to contain M. tuberculosis within a caseous granuloma, which offers the macrophages inside an opportunity to kill the raining organisms if the necrotic area is small enough.

____________________ takes the form of a budding yeast with a thick capsule. This yeast also grows abundantly in soil containing *bird (pigeon) droppings*. However, this fungus tends to cause disease (meningoencephalitis and pulmonary disease) in the immunocompromised.

Cryptococcus neoformans

Inactivated toxins (toxoid) vaccines include the _______________ and _________________ vaccine. Toxoid vaccines are useful when a toxin is the main cause of disease morbidity and mortality, as they allow the body to develop neutralizing antibodies agains the toxin.

Diphtheria ; tetanus

_________________________ results in poor leukocyte adherence and transmigration through the vasculature. Affected patients present with recurrent *skin* and *mucosal* infections

Leukocyte adhesion deficiency

____________________ is a metabolite of arachidonic acid. Its main function is to stimulate neutrophil migration to the site of inflammation.

Leukotriene B4

uWORLD: A 54 yo Caucasian male is hospitalized with spiking fevers and productive cough. He was diagnosed with right lower lobe pneumonia one week ago and received a short course of oral antibiotics, but his condition has failed to improve since then. Chest X-ay shows a round density with an air-fluid level in the lower lobe of the right lung. Which of the following contributed most to the observed lung lesion in this patient?

Lysosomal content release by macrophages > Pulmonary abscesses are local suppurative collections within lung parenchyma that result in necrosis of the surrounding lung tissue. If the abscess cavity communicates with an air passage, the semiliquid exudate within will partially drain, creating an air-containing cavity that can be identified on chest radiograph. > Histologically, suppurative destruction of the lung parenchyma within the abscess cavity is seen. This destruction occurs to a large extent secondary to the release of lysosomal enzymes by neutrophils and macrophages. The lysosomal enzymes severe to digest the offending pathogens and tissue debris, as well as to chemotactically summon additional neutrophils or macrophages to the area. Occasionally, however, the enzymes will also damage the surrounding parenchyma, setting the stage for abscess formation. > Tissue damage and resultant abscess formation is primarily caused by lysosomal enzymes release from neutrophils and macrophages.

uWORLD: A 5 yo boy is brought to the ER with breathing difficulty, dysphagia, drooling and fever. His temperature is 103.0 F. WBC count is 23,000 with many band forms. Laryngoscopy in the operating room shows a swollen and cherry-red epiglottis. This patine most likely:

Missed vaccination > Rapidly progressing fever, severe sore throat, drooling and progressive airway obstruction potentially accompanied by stridor are the presenting symptoms of acute epiglottitits. This illness is most commonly caused by H. influenzae type b, but the Hib vaccine has dropped the incidence of this disease considerably. H. influenzae type b can still cause disease in unimmunized or improperly immunized patients as well as fully immunized patients in some cases.

____________________ is a cell wall component and major virulence factor of Mycobacterium tuberculosis. It protects M tuberculosis from being killed by macrophages and stipulates granuloma formation.

Trehalose dimycolate

A 32 yo woman comes to the office with worsening SOB. She has the following arterial blood results: PaO2: normal % saturation (SaO2): normal Oxygen content: low Which of the following is the most likely cause of these results?

Chronic blood loss > Anemia is characterized by decreased hemoglobin concentration in the setting of normal SaO2 and PaO2.

___________________ pneumoconiosis can present with exertion dyspnea and nodular interstitial opacities on chest x-ray. Histology of nodal and perilymphatic lung tissue shows accumulations of coal dust-laden macrophages (coal macules).

Coal worker's

_________________ also causes disseminated mycosis in immunocompromised patients. In tissue sections, it is visualized as large, thick-walled spherules containing endospores.

Coccidiodies immitis

______________________ typically causes atypical penuomina characterized by a persistent nonproductive cough, pharyngitis, ear pain, and constitutional symptoms (eg. fever, headache, malaise). Chest x-ray classically reveals a diffuse interstitial infiltrate.

Mycoplasma pneumoniae

____________________ is an enzyme found in neutrophil azurophlic granules the raids in intracellular killing by catalyzing the production of hypochlorite (bleach) from hydrogen peroxide and chlorine

Myeloperoxidase

uWORLD: A 3 yo boy is brought to the office with 5 days of productive cough and fever. Examination shows a tachypneic, ill-appearing child with rales over the right lower lung. Chest x-ray reveals right lower lobe pneumonia. the boy is admitted to the hospital for intravenous antibiotics. Review of his medical history shows 4 prior pneumonia's, and his weight is at the third percentile. Further testing demonstrates high sweat chloride content, and genetic sequencing shows a mutation in a transmembrane protein. Which of the following best describes the dysfunctional transmembrane protein causing this patient's disease?

ATP-gated > Cystic fibrosis (CF) is an autosomal recessive disorder due to mutations (eg. deltaF508) in the CF transmembrane conductance regulator (CFTR) protein. The channel pore *opens* after binding of *2 ATP molecules*, allowing transport of chloride ions down the electrochemical gradient. > Cystic fibrosis is an autosomal recessive disease disease caused by mutations in the CF transmembrane conductance regulator (CFTR) gene. The CFTR protein is a transmembrane ATP-gated chloride channel. Defects in CFTR result in thick, plugged mucous and elevated sodium and chloride levels in sweat.

_________________ is a macrolide antibiotic that acts by inhibiting the material 50S ribosomal subunit as do all the macrolide antibiotics, chloramphenicol, and linezolid.

Azithromycin

uWORLD: A 6 yo boy playing in a dusty field in windy weather inhales many small particles that become lodged in his terminal bronchioles. Which of the following respiratory components is most important in learning these particles?

Ciliated cells > Most inhaled particles that lodge in the bronchial tree are moreover via proximal transport by ciliated epithelial cells (mucocillary clearance). Mucus-secreting cells are present to the level of the larger bronchioles, after which *club cells* become the prominent secretary cell type.

_________________ inhibits fungal cell mitosis at metaphase. It is indicated for the treatment of dermatophytoses.

Griseofulvin

____________________ is endemic to the Mississippi and Ohio River and found in bird and bat droppings. Patients with this often have a history of cleaning bird coops or caving.

Histoplasma capsulatum

_________________ is characterized by poorly regulated rapid eye movements (REM) sleep. It frequently causes excessive daytime sleepiness, but patients also suffer from cataplexy (sudden and transient episode of muscle weakness accompanied by full conscious awareness) , sleep attacks, hypnoagonic/hypnopompic hallucinations, and sleep paralysis.

Narcolepsy

______________________ usually arises from the major bronchi. On chest imaging, it is seen as a hilar mass. Pathology shows flat, oval-shaped cells with scant cytoplasm and hyper chromatic nuclei. Neuroendocrine markers such as *chromogranin* and *synaptophysin* are usually positive.

Small cell carcinoma

Coagulase is an enzyme produced by ________________ that activates prothrombin, resulting int he conversion of fibrinogen to fibrin. This process leads to fibrin-coating of the organism and resistance to phagocytosis.

Staph aureus

Live attenuated bacterial vaccines include the _____________ and __________ vaccine. Live attenuated vaccines are more effective at inducing immunity because the organism are actively growing with the inoculated host. However, attenuated vaccines can revert to virulent strains and can cause serious complications in immunocompromised hosts.

Tuberculosis (BCG) ; oral typhoid

uWORLD: A 34 yo immigrant from Eastern Europe presents to your office with a three-month history of productive cough, night sweats and low-grade fever. Sputum cultures grow budding yeast that form germ tubes at 37 C. The most likely site of this organism before entering the sputum is ?

oral cavity > Canaida give rise to tree hyphae, termed "Germ tubes", when incubated at 37 C for 3 hours. Morphologically, all Candida fungi are yeasts, seen in tissue sections as single cells with *pseudohyphae*. > Candida albicans is the most common opportunistic mycosis. It is also a frequent colonizer of human skin and mucous membrane. (Candida contributes to the normal flora off skin, mouth, vagina, and intestine).

uWORLD: A county hospital experiences an outbreak of methiclin-resistant Staphylococcus aureus (MRSA) would infections. You suspect that many staff members are carriers of the bacteria. Which of the following would you expect to be the most commonly MRSA-populated site in staff members? (Hands, axilla, anterior nares, perineum, oropharynx)

Anterior nares > In the general population at any given time, 25-30% of individuals have nasal colonization with Staphylococcus aureus. The *anterior nares* are the most common site of colonization for both methicllin-sensitve and methicillin-resistant Staph aureus.

uWORLD: A 50 yo non-smoking female is found to have a round lesion in the right upper lobe of her lung. Transthoracic biopsy is scheduled to obtain tissue for histological examination. Which of the following would favor the diagnosis of hamartoma in this patient?

Cartilage tissue in the biopsy sample > Hamartomas are the most common benign lung tumors. They present as asymptomatic peripherally located "coin lesion" in patients 50-60 years old. These tumors are composed of disorganized cartilage, fibrous and adipose tissue.

*Vancomycin* is active only against ____________________ bacteria and thus does not provide good coverage against anaerobes.

Gram-postive

____________________ is a systemic vasculitis of small and medium-sized arteries. It presents with involvement of upper respiratory tract (sinusitis, nasal obstruction, epistaxis, otitis), lung (hemoptysis) and kidney *rapidly progressing glomerulonephritis). Presence of cANCA is typical.

Granulomatosis with polyangiitis (Wegener's)

uWORLD: A 62 yo woman comes to the physican with sought and dyspnea. She electorates copious amounts of pale tan-colored fluid. Chest x-ray reveals a pulmonary infiltrate that is subsequently biopsied. Histological examination shows columnar mucin-secretin cells that line the alveolar spaces without invading the storm or vessels. This patient's condition is best categorized under which of the following disease processes?

Malignant neoplasm > *Adenocarcinoma in situ* (formerly known as bronchioloalveolar carcinoma) is one of the major subtypes of lung subtypes of lung adenocarcinoma, the most common type of lung cancer in the U.S. > The tumor arises form the alveolar epithelium and is located at the periphery of the lung. > It is considered a pre invasive lesion characterized by *growth along interact alveolar septa* without vascular or stromal invasion. > Microscopic examination reveals *well-differentiated, dysplastic columnar cells* with or without intracellular mucin (compare to normal lung). The tumor has a tendency to undergo erogenous spread (along the airways) and can progress to invasive disease if not resected.

uWORLD: Bacteria isolated from the lung tissue of a 32 yo Caucasian male fail to depolarize with HCL and alcohol after staining carboifuchsin. Which of the following cell wall components is most likely responsible for this atoning phenomenon?

Mycolic acid > The acid-fast stain identifies organism that have mycelia acid present in their cell walls, including Mycobacterium and some Nocardia species. > Acid-fast staining is carried out by applying an aniline dye (eg, carbolfuchsin) to a smear and then decolorizing with acid alcohol to reveal whether the organisms present are "acid fast".

uWORLD: A 52 yo man is being evaluated for persistent nonproductive cough and exertion dyspnea that has progressed over the past year. Lately he has had difficulty accomplishing normal daily activities. The patient has no significant past medical history and takes no medications. He does not use tobacco, alcohol, or illicit drugs. Spirometry shows a forced vital capacity (FVC) that is 40% of the predicted value and a FVC1 to FVC ratio of 87%. A CT guided lung biopsy is performed, and histopathology sows dense fibrosis, fibroblast proliferation, and cyst formation, which are most prominent in the subpleaerula regions. Which of the following is the most likely diagnosis?

idiopathic pulmonary fibrosis > This patient presenting with slowly progressive external dyspnea and dry cough, a *resrictive profile* on pulmonary function testing (PFT), and international fibrosis with cystic air space enlargement likely has *idopathic pulmonary fibrosis (IPF). IPF is a type of idiopathic intersitital pneumonia, a group of conditions that are classified based on acuity and smoking history. > The pathologic findings of IPF are termed "usual interstitial pneumonia (UIP)" and show patch involvement with dense *fibrosis* and fibroblastic foci. Alveolar wall collapse leads to formation of *cystic space (honeycombing)* lined by hyper plastic type II pneumocytes or bronchiolar epithelium (honeycomb fibrosis). These findings are more predominant in the sub pleural and paraseptal spaces.

____________ is a fast-acting, depolarizing neuromuscular blocking agent used for rapid-sequence intubation that causes equal reduction of all 4 twitches during train-of-four stimulation (phase I blockade). Prolonged administration of succinylcholine or use in patients with abnormal plasma cholinesterase activity causes transition to a phase II (non depolarizing) block, seen as a progressive reduction in each of the 4 twitches.

Succinylcholine

___________________ causes *hereditary angioedema* that can be trigged by transfusions. Affected patients have angioedema but do NOT have urticaria or wheezing.

C1 inhibitor deficiency

The pulmonary vascular bed is relatively unique in that tissue hypoxia results in a _________________ response. Such response. Such hypoxic response occurs in the small muscular pulmonary arteries to divert blood flow away from under ventilated regions of the lung toward better-ventilated areas.

vasoconstriction

Prenatal supplements that should be taken include? _________, _______, _________, ___________

> Folic acid in prenatal vitamins can help prevent neural tube defects. > Iron - for hemoglobin synthesis > Calcium - for bone growth > Iodine - for thyroid function

uWORLD: Drug X, a laboratory -synthesized antimicrobial agent, is a D-alaine-D-alanine analog that block bacterial peptidoglycan cross-linking and resists degradation by bacterial enzymes. Antimicrobial diffusion disks with drug X are added to several agar plates, each which contains colonies of a single organism with the appropriate growth medium. The plates are then inverted and incubated for 18 hours alongside control plates with the same organism but no antimicrobial disks. The control plates all show organism growth. In the plates with the diffusion disks, resistance to drug X is detmerined by measuring the zone of complete growth inhibition around the disk. Which of the following bacteria is most likely to be resistant to drug X? (Actinomyces israelii, Borrelia burgdorferi, Helicobacter pylori, Mycoplasma hominis, Pasteurella mutlocida)

> The new antibiotic drug X inhibits synthesis of the *peptidogylcan cell wall* found in both gram-positive and gram-negative orngaims. This cell wall is essential for survival of these organism because it acts as a permeability barrier and protects them from destruction by osmotic stressors. > All organisms in the Mycoplasma genus, including Ureaplasma, lack peptidoglycan cell walls and are therefore resistant to agents that attack the peptidoglycan cell wall such as penicillins, cephalosporins, carbapenems, and vancomycin. > Mycoplasma infection can be treated with anti-ribosomal agents (Eg. tetracycline, macrocodes)

uWORLD: A 55 yo man comes to the office due to malaise and cough over the past 2 months. He describes yellow sputum production with occasional streaks of blood. The patient smokes a pack of cigarettes daily and has a history of alcohol abuse with prior episodes of binge drinking. Temperature is 98.8 F. Examination shows poor dentition with dental caries, gingivitis, and enlarged submandibular lymph nodes. Coarse rhonchi are heard during auscultation of the right lung. Chest CT scan reveals an extensive right lung consolidate process with air bronchograms. Bronchoscopy is performed, and a lung bossy specimen shows

Actinomycosis is a slowly progressive ideas caused by gram-positive aerobic bacteria. The organisms typically colonize the mouth, colon, and vagina and can be found in dental caries as well as the margins of gums in patients with poor dentition. Actinomyces infection most frequently leads to the formation of cervicofacial abscesses, but systemic infection can develop anywhere in the body when the mucosa is disrupted. > *Pulmonary* actinomycosis (relatively rare) develops most commonly by *aspiration*, which often leads to lower lobe consolidation with air bronchograms. > Diagnosis is made by identifying the bacteria with unique *filamentous, branching patterns* and the characteristic *sulfur granules*, which are formed by calcified mycelial fragments. > Sulfer granules process appear yellow; however, hematoxylin and eosin staining gives them an amorphous basilica (purple/blue) appearance under light microscopy.

uWORLD: A 55 yo woman comes to the physician because of a persistent cough and recent, unintentional weight loss. She has never smoked and has no history of exposure to industrial pollutants. Physical examination reveals decreased breath sounds and dullness to percussion at the left lung base. Imaging suites show an irregular mass in the lower lobe of her left lung and a left-side pleural effusion. A diagnostic thoracocentesis is performed and he aspirated fluid is sent for cytological evaluation. If a malignancy is diagnosed it is most likely to be?

Adenocarcinoma > Adenocarcinoma is the most common lung cancer in the vernal pollution. It is also the most common subtype in women and nonsmokers. It is locked peripherally and consists of tumor cells that form glandular or papillary structures. > In constrast, squamous cell carcinoma and small cell carcinoma have a strong association with smoking.

___________________________ is an autosomal recessive disorder that is the scones most common cause of severe combined immunodeficiency (SCID). This condition results in a profound decrease in B- and T-lymphocytes number with variable immunoglobulin deficiencies.

Adenosine deaminase deficiency

uWORLD: A 35 yo man comes to the ED with recurrent hemotphysis, weight loss, and low-grade fevers for several weeks. he says he lost 7 kg over the past 3 months. He recently emigrated from Central America. The patient's temperature is 100 F, blood pressure is 110/70, pulse is 78, and respirations are 18. A chest x-ray demonstrates right lung apical infiltrative and cavitary lung lesions. Sputum cultures grow acid-fast bacilli. he is placed in respiratory isolation and started on antibiotic therapy. The cavitary lesions seen in this patient most likely formed through which of the following pathogenic mechanisms?

Aggregation of activated leukocytes. > Mycobacterium tuberculosis infection characteristically demonstrates granulomatous inflammation with caseous necrosis. Granuloma formation assets in disease containment and occurs mainly through an interaction among macrophage, multinucleate giant cells, and CD4 T lymphocytes. Extensive macrophage activation can also result in collateral tissue damage, resulting in caseous necrosis with formation of cavitary lung lesions.

Patients with asthma are at risk for developing an allergic reaction to A. fumigates called _____________________. Signs and symptoms include cough, dyspnea, wheezing, fever, and migratory pulmonary infiltrates.

Allergic bronchopulmonary aspergillosis

uWORLD: A 55 yo man comes to the ED due to sudden onset of dyspnea. He is a truck driver and just returned from a long trip. The patient's medical history is significant for hypertension, hyperlipidemia, diabetes mellitus type 2, and chronic kidney disease. Blood pressure is 110/70 mmHg and pulse is 110/min. Physical examination shows a moderately overweight man with tachypnea. Lungs are clear on auscultation. ECG shows sinus tachycardia. His serum creatinine is elevated. Ventilation/perfusion scanning is ordered. Which of the following findings will be most helpful to confirm the suspected diagnosis in this patient?

An area of perfusion defect without ventilation defect *Pulmonary embolism* (PE) should be suspected in this patient with a prolonged period of immobility (long trip) who now has acute onset of dyspnea and tachypnea, a normal lung examination, and sinus tachycardia on ECG. > In most patients, CT angiography, which requires contracts administration, is the diagnostic test of choice. > However, contrast used in CT studies should be avoided in patients with renal insufficiency (elevated creatinine) due to the increased risk of contrast-indcued nephropathy, in such patients, a *ventilation/perfusion scan* (V/Q) is the peered diagnostic study.

uWORLD: A 12 yo boy is brought to the ED because of difficulty breathing, wheezing, and nonproductive cough. He has visited the ED seven times over the past year because of similar symptoms. Physical examination reveals decreased bilateral breath sounds, prolonged expiration, and end-expiratory wheezes. Chest x-ray shows hyperinflated lungs with no infiltrates. He is treated with inhaled beta-agonists and corticosteroids. He significantly improves and is discharged on tapered-dose oral corticosteroids. He comes to the ED 2 weeks later in severe respiratory distress with similar symptoms and requires endotracheal intubation. Which of the following therapies will most likely decrease the likelihood of such event happening in the future?

Anti-IgE antibodies > Omalizumab is an effective and acceptable add-on therapy for patients with severe allergic asthma. it has been shown to be effective in reducing dependency on both oral and inhaled steroids. NOTE: > H1 receptor antagonists are effective in the treatment of chronic urticaria and allergic symptoms but not effective in the treatment of asthma.

uWORLD: An 18 yo man comes to the ED with sudden-onset right-sided chest pain and dyspnea. The patient was at home watching a football game on television when his symptoms abruptly started. He now has pain with depth breaths. He has no the medical problems and takes no medication. The patient has smoked a pack of cigarettes daily for the past 2 years. His respirations are 24/min. Physical examination shows a thin, tall patient in acute distress. The right side of the chest is hyper resonant to percussion and lacks audible breath sounds. What condition most likely led to this patient's presentation?

Apical sub pleural blebs > This young male patent with sudden-onset unilateral chest pain, dyspnea, and absent breath sounds on examination likely has *primary spontaneous pneumothorax (PSP). PSP is no traumatic and is found in patients without preexisting pulmonary disease (eg. cystic fibrosis). It occurs when a large change in the alveolar or intrapleural pressure results in a break in the visceral pleura and subsequent trapping of air between the parietal and visceral spaces. The superficial alveoli in the apices experiences greater pressure changes (due to the weight of the lungs pulling down on the apical tissue), predisposing them to the formation of *subpleural blebs* The blebs then can spontaneously rupture through the visceral pleura, frequently while the patient is at rest. > Tall, thin males around the age of 20 are most commonly affected. Although the most important risk factor is *smoking*, taller individuals also appear to be at higher risk due to more negative intrapleural pressure in the lung apices.

uWORLD: A 64 yo man is brought to the ED after a motor vehicle accident where his chest hit the steering wheel. His termpature is 98 F, blood pressure is 132/78, pulse is 76, and respirations are 14. The patient has no known chronic medical problems and takes no medication. Physical examination reveals mild tenderness over the lower right chest. Imaging of the neck and chest shows no fractures of dislocations. However, the chest x-ray reveals pleural thickening and calcifications along the posterolateral middling regions and diaphragm. There is also a small right-sided pleural effusion. This patient most likely has a history of exposure to which of the following agents? (Asbestos, Beryllium, Coal dust, Nitrous dioxide, organic dust, silica)

Asbestos > This older adult patient, who is incidentally found to have *pleural thickening* with calcification of the posterolateral middling zones and diaphragm, likely has *asbestos-related* pleural *disease*. These *calcified lesions *pleural plaques)* are one of the hallmarks of asbestos exposure and usually affect the parietal pleura. > Asbestos-realted pleural disease presents with pleural thickening, calcified lesions (pleural plaques) of the posterolateral middling zones and diaphragm, and occasionally benign pleural effusions. Many patients are asymptomatic despite visible disease on imaging.

____________________ is a type of pneumoconiosis that can present with dyspnea on exertion, but the chest x-ray is more likely to reveal an interstitial pattern of involvement most prominent in the lower zones. *Pleural plaques* may also be noted. Histology shows ferruginous bodies - fusiform rods with a translucent asbestos center and an iron-containing coating.

Asbestosis

uWOLRD: A 34 yo woman comes to the office for evaluation of recurrent transient pulmonary infiltrates. The patient has history of bronchial asthma and has had several exacerbations over the past years, particularly during the winter months. She is currently asymptomatic. She has no other medical problems and has never traveled outside the U.S her medications include albuterol as needed and medium-dose inhaled glucocortocids. Temp is 98.8 F, physical examination is unremarkable. Complete blood count shows eosinophil. A chest Ct scan reveals proximal bronchiectasis. The patin's condition is most like related to colonization with which of the following?

Aspergillus fumigateurs > *Aspergillus fumigateurs* is a low virulence fungus that generally does not cause significant infections except in immunocompromised or deliberated patients. It may, however, colonize the bronchial mucosa. > Patients with asthma or cystic fibrosis in particular may develop an allergic *hypersensitivity* reaction to the fungus. > The result is *allergic bronchopulmonary aspergillosis (ABPA), which occurs in 5%-10% of corticosteroid-depndent *asthmatics*. > Patients with ABPA have very high serum IgE levels, *eosinophilia*, and IgE plus IgG serum antibodies to Aspergillus. > There is intense airway inflammation and mucus plugging with exacerbations and remissions. Repeated exacerbations may produce transient pulmonary infiltrates and proximal *bronchiectasis*

___________________ can colonize old lung cavities (eg. those formed by tuberculosis) to form a "fungal ball". Symptoms include cough, dyspnea, and hemoptysis.

Aspergillus fumigatus

A sample of contaminated moist soil is heated to 100 C for 15 minutes. Which of the following bacteria is most likely to be removed from the soil sample following heat exposure?

Bacillus anthracis > Spore-forming bacteria can survive boiling. Bacillus anthracis and members of the genus Clostridium are potentially pathogenic bacteria found in the soil and capable of forming spores.

uWORLD: A 56 yo man comes to the office after 2 episodes of low-volume hemoptysis. He describes it as "streaks of blood" in his sputum. The patient has had a chronic cough over the last several years, most prominently in the morning. He has also had several recent respiratory infections. The patient has smoked a pack of cigarettes daily for the last 40 years. Chest x-ray shows hyper inflated lungs but no infiltrates or masses. Bronchoscopy is performed and several suspicious foci of bronchial mucosa are biopsied. Microscopy of the biopsy sample reveals areas of strafed squamous epithelium. The pathogenesis of this patient's microscopic findings is most similar to which of the following conditions? (Barret Esophagus, Cervical cancer, Hypertrophic cardiomyopathy, Interstital cystitis, Minimal change disease)

Barrett esophagus > Squamous metaplisa is a reversible, adaptive response to chronic irritation, such as smoking. The normal columnar epithelium is replaced by squamous epithelium, which is more resistant to irritation but has reduced mucocilarly clearance. Metaplasia also occurs with Barrett esophagus, in which esophageal squamous epithelium is replaced by columnar epithelium in response to chronic acid exposure.

_________________ is a type of pneumoconiosis that may present with dyspnea and ill-defined nodular or irregular opacities on chest x-ray. Histology reveals noncaseating epithloid granules without obvious, associated particles.

Berylliosis

uWORLD: A 46 yo man comes to the physician due to 3 days of fever, shortness of breath, pleuritic chest pain, and cough productive of green sputum. He has smoked a pack of cigarettes daily for 20 years. On examination, there are crackles at the base of the left lung. Chest x0ray reveals a left lower-lobe consolidation. Microscopic analysis of a sputum sample shows gram-positive, lancet cocci in pairs. Which of the following characteristics are these bacteria likely to demonstrate?

Bile solubility > Streptococcus pneumoniae are gram-positive, alpha hemolytic, optochin-sensitve, bile soluble diplococci. > Virdans group stretopoccic are also alpha hemolytic, but they are optochin-resistant and bile insoluble. > Streptococcus pyogenes (Group A streptococcus) appear as gram-positive cocci in chains with bacitracin susceptibility.

uWORLD: A 32 yo Caucasian male presents with low grade fever cough, and generalized malaise. The patient has otherwise been healthy and has no significant medical history. His past surgical history includes tonsillectomy and lympanstomy tube placement at the age of 8. Careful questioning reveals that the patient recently came back from an "outdoor adventure" trip in the Great Lakes, and began feeling sick only after returning. Microscopic examination of a KOH preparation of an exudate specimen reveals a large yeast cell with a single bud. When incubated in a specific medium at 75.2 F, a multicellular structure with branching, tubular cells grows. The patient is most likely infected with which?

Blastomyces dermatitidis

uWORLD: A 49 yo man comes to the office due to 4 months of cough and weigh loss. His medical history is otherwise unremarkable and he takes no medications. The patient is a farmer who lives in Mississippi. He does not use tobacco, alcohol, or illicit drugs. His temperature is 100.2 F. Examination shows right lobe crackles. Chest x-rays reveals a pulmonary infiltrates in the right upper lobe. Bronchoscopy with transbronchial biopsy shows granulomatous inflammation. HIV testing is negative. Which of the following is most likely causing this patient's symptoms?

Blastomyces dermatitidis > *Blastomyces dermatitidis* is a dimorphic fungus that can cause pulmonary infections in *immunocompetent* people and should be considered in individuals form an endemic area (states adjacent to and east of the Mississippi and Ohio river valleys; some overlap with *Histoplasmosis). > *Pulmonary* blastomycosis is characterized by *granuloma* formation. > Dissemination (most commonly to skin and bones) occur in *immunocompromised* individuals. > Itraconazole is the preferred treatment for blastomycosis.

uWOLRD: A 48 yo man comes to the physician with a 6 month history of cough and fatigue. He also has experienced malaise and weight loss. He does not recall any sick contacts. The patient is a carpenter. He smokes 1 pack of cigarettes daily. His temperature is 98.8 F. Examination shows left lower lobe crackles. HIV testing is negative. Chest x-rays reveals a pulmonary infiltrate in the lower lobe of the left lung. Results are from a sputum potassium hydroxide preparation. What is most likely caused of this patient's condition?

Blastomyces dermatitidis > Blastomyces dermatitidis is a dimorphic fungus seen in tissue as round or oval yeasts with thick walls and broad-based budding. It is endemic in the southeastern U.S (states east of the Mississippi River). The lungs are the primary site of involvement, and the skin and *bone (caused by lytic lesions)* are the major sites of dissemination. > Histology shows a typical *large yeast* with a single, *broad-based bud*.

uWORLD: A 56 yo man comes to the physician for a follow-up appointment. His medical problems include HTN and osteoarthritis for which he takes amlopdine daily and ibuprofen as needed. He was employed as a construction worker for over 30 years and recently received a latter from his employer stating that he had significant exposure to asbestos early in his cancer. The patient is now concerned about his risk of developing cancer. Which malignancy is likely to develop? (Acute leukemia, bronchogenic carcinoma, gastric carcinoma, malignant mesothelioma, transitional cell carcinoma)

Bronchogenic carcinoma > Asbestos is naturally occurring mineral with insulating properties used in shipbuilding, construction, and textile industries. Inhalation of fine asbestos fibers in occupational and non-occupational settings causes epithelial cell injury, activation of macrophages, and chronic interstitial inflammation and fibrosis. > Patients with a long history of asbestos exposure are at risk for developing asbestosis, plural disease, and malignancies such as bronchogenic carcinoma and mesothelioma. *Bronchogenic carcinoma* is the most common malignancy in this population, followed by *mesothelioma*

Deficiency of the ________________________ causes increased susceptibility to encapsulated bacteria (eg. Streptococcus pneumoniae, Haemphilus influenzae, Neisseria meningitidis) and also predisposes to developing systemic lupus erythematous.

C1 complement component

uWORLD; A 35 yo African American is being evaluated for exptional dyspnea and dry cough. She has no significant pas medical history and takes no medication. She works as a bird keeper part time at a local zoo. Chest x-ray shows bilateral hilar adenopathy and reticular pulmonary infiltrates. Serum ACE levels and calcium levels are elevated. PPD testing is negative. Bronchoscopy is arranged to help determine the diagnosis. Which of the cell times is most likely to predominate in this patient's brocnhoalvelar lavage fluid?

CD4+ lymphocytes > CD4+ T helper cells are the predominant crypt of lymphocyte found in sarcoid granulomas. Intralveoalr and interstitial accumulation of CD4+T cells i

A 23 yo male runs 5 miles. Which do you expect to increase at the peak of his exertion. (CO2 content, O2 content, CO2 content in the mixed venous blood, pH of the arterial blood, physiologic dead space"

CO2 content in the mixed venous blood > During aerobic exercise, increased skeletal muscle CO2 production increases the PCO2 of mixed venous blood. Homeostatic mechanism maintain arterial blood gas levels and arterial pH near the resting values.

A 3 yo boy who recently immigrated to the US is brought to the pediatric emergency department with fever, malaise, and a swollen right knee. He is hypotensive and tachycardic. His past medical history is significant for a recent episode of otitis media and several vaccinations that are not up to date. Arthrocentesis of his right knee shows cloudy synovial fluid. Gram stain of the aspirate reveals pleomorphic, Gram-negative cocobacilli. Cultures performed on a blood agar plate supplemented with a disk containing hematin and NAD+ grow colonies only near the disk. The pathogenicity of the organism responsible for this patient's condition is most likely related to which of the following virulence factors? (Capsule, cytotoxic exotoxin, Fimbiae, Hemolysins, hyaluronidase)

Capsule > Haemophilus influenzae is a blood-loving organism that requires both X factor (hematin) and V factor (NAD+) to grow. The phatogenicity of H influenzae depends on the presence of an antiphagocytic polysaccharide capsule. The type B strain is the most invasive and virulent and has a capsule consisting of liner polymer of polyribitol phosphate. > Unecapsualted (nontypable) H influenzae is part of the normal flora and causes primarily local infections.

uWORLD: It has been determined in healthy volunteers that the chloride content of erythrocytes is much lower in arterial blood than in venous blood. The action of which of the following is most likely responsible for the observed difference?

Carbonic anhydrase > Carbonic anhydrase activity within erythrocytes form bicarbonate from CO2 and water. Many of the bicarbonate ions diffuse out of the RBC into the plasma. To maintain the electrical neutrality chloride ions diffuse into the RBC to take their place. This process is called "chloride shift", and it is the principal cause of high RBC chloride content in venous blood.

uWORLD: A 56 yo man comes to the office for chronic cough and fatigue. The patient has smoked 2 packs of cigarettes per day since age 18. Physical examination shows cyanosis and expiratory wheezes scattered throughout the lungs. During evaluation for long-term oxygen therapy, his respiratory rate decrease shortly after beginning nasal annular oxygen supplementation. The patient's reduced respiratory rate is most likely caused by a sudden decrease in stimulation of which of the following sensory reactors?

Carotid bodies > PaCO2 is the major stimulator of respiration in healthy individuals; even a slight increase in PaCO2 results in increased pulmonary ventilation. In patients with chronic obstructive pulmonary disease, response to PaCO2 is blunted and hypoxemia becomes an important contributor to the respiratory drive. Peripheral chemoreceptors are primarily responsible for sensing arterial PaO2 and can be suppressed with oxygen administration.

uWORLD: A 4 yo boy is being evaluated due to severe and recurrent infections. He has been hospitalized 3 times with pneumonia since birth. He has also had multiple skin infections that have required treatment with antibiotic and anitfungal agents. The patient's neutrophils fail to turn blue following exposure to nitroblue tetrazolium. Which of the following substances is most likely produced by the microorganisms responsible for this patient's function?

Catalase > The failure of this patient's neutrophils to turn blue on *nitroblue tetrazolium testing* is characteristic of *chronic granulomatous disease, a condition must often caused by an X-linke mutation affecting *NADPH oxidase*> This enzyme normally functions within activated phagocytes to produce *reactive oxygen species that act directly as antimicrobial agents and also activate *granule proteases* (eg, elastase, cathepsin G) present in phagosomes. > Inactivating mutations in NADPH oxidase cause *impaired intracellular killing* by neutrophils and macrophages, leading to recurrent bacterial and fungal infections. > These infections are usually caused by *catalase-postive* organism that can destroy the hydrogen peroxide produced by their own metabolic activity.

uWORLD: Gram-postive bacteria are inoculated under the skin of experimental animals and then the infection is treated with antibiotics. Bacteria isolated from the injection site several days later assume a spherical configuration when placed in an isotonic solution and disintegrate rapidly when placed in a hypotonic solution. Which of the following antibiotic was most likely used in this experiment? (Chloramphenicol, azithromycin, ciprofloxacin, cefuroxime, doxycycline)

Cefuroxime > Under normal circumstances, Gram-postive organism would not be destroyed by variations in tonicity within a certain range due to their intact peptidoglycan cell wall. > Cefuroxime is a second generation cephalosporin. > Cephalosporins are beta-lactam antibiotics and are related to penicillins. Vancomycin is another example of an antibiotic that inhibits cells all synthesis.

uWORLD: A 14 yo boy is brought to the ED by his mother after he develops a sudden rash. He was trying to retrieve a baseball that had rolled under a log when he got stung by something. BY the time he told his mom, his body was covered in wells. His mother gave him diphenhydramine, but the patient's face started to swell and he was having a hard time swallowing. The patient's blood pressure is 70/50 and HR is 120. Physical examination shows erythematous, raised plaques over the tank, extremities, and face. Lung auscultation reveals bilateral expiratory wheezes. The mother is not aware of the child having any similar reaction in the past. Which of the following most likely triggered this patient's condition?

Cell surface-bound antibody bridging by antigen > Type I hypersensitivity reactions are mediated by th interaction of allergen with preexisting IgE bound to basophils and mast cells. The facilitates cross-linking of the surface IgE molecules that signals the cell to degranulate releasing chemical mediators (eg, histamine, heparin). These agents are responsible for the immediate signs and symptoms of allergy, from a local wheal and flare to life-threatening anaphylaxis.

____________________ breathing describes cyclic breathing in which apnea is followed by gradually increasing then decreasing tidal volumes until the next apnea period. It is common seen in the setting of *advanced congestive heart failure*

Cheyne-Stokes > Patients with CHF have chronic hyperventilation with hypocapnia, which induces apnea during sleep when the partial pressure of carbon dioxide (PaCO2) falls below a certain level ("apnea threshold")

__________________ is an antibiotic in the flurorquinolone class of antibiotics. The fluroquinolones act by inhibiting bacterial DNA gyrate thereby leading to bacterial DNA damage and bacterial death. These agents have no effect on the cell wall.

Ciprofloxacin

uWORLD: A 78 yo woman is borough to the hospital due to 2 days of nausea, vomitting, abdominal pain, and headache. The patient has a history of chronic obstructive pulmonary disease for which she takes theophylline. She was recently hospitalized for a stroke, which was complicated by a UTI and delirium. The patient appears agitated and restless. Her temperature is 98.8 F, blood pressure is 110/70, pulse is 118, and respirations are 22. She has a fine hand tremor on examination. Chest escalation reveals mild expiratory wheezing. Her serum theophylline levels are markedly elevated. Treatment with which of the following agents most likely precitpated her current symptoms? *Amoxicllin, cephalexin, ciprofloxacin, haloperidol, nitrofurantoin)

Ciprofloxacin > This patient was most likely treated with *cirpofloxacin* for her UTI infection and experienced subsequent *theophylline toxicity*. Theophylline is an adenosine receptor antagonist and phosphodiesterase inhibitor that is sometimes used as an alternate therapy for asthma and COPD. It causes *bronchodilation* primarily by increasing intracellular cyclic AMP levels *similar to beta-adrenerig agonists) and also has mild anti-inflammatory effects. > It is predominantly metabolized by the hepatic cytochrome oxidases. Inhibition of these enzymes by concurrent illness (eg. infection with fever) or drugs (eg. cimetidine, ciprofloxacin, macolids, verapamil) can raise serum concentrations and cause toxicity *eg, seizures, cardiac arrhythmias)

uWORLD: A 45 yo man comes to the ED due to several days of cough and fever. His temperature si 101.2 F, blood pressure is 117/76, pulse is 92/min, and respirations are 18/min. Physical examination of the right lung reveals occasional rales and decreased breath sounds at the base. A chest x-ray demonstrates a lung infiltrate, hilar adenopathy, and right-sided pleural effusion. Lung tissue obtained from this patient showing endospores. Was is most likely the cause of this patient's condition?

Coccidioides immitis > Coccidioides immitis infection can be asymptomatic or it can cause pulmonary disease ranging from flulike illness to chronic pneumonia. It causes disseminated disease in immunocompromised patents. Spherules containing endospores are found in the tissue samples.

uWORLD: A 42 yo previously healthy man comes to the ED due to several hours of dyspnea. He is long-haul truck driver and became acutely short of breath during a 36 hour trip. The patient smokes a pack of cigarettes daily and drinks a 6 pack of beer on the weekends. Pulmonary examination reveals vesicular breath sound bilaterally with the overlying areas resonant to percussion. CT pulmonary angiogram shows right lower lobe pulmonary artery occlusion consistent with pulmonary embolism but no evidence of pulmonary infarction. Which of the following is the most likely reason for the lack of tissue necrosis in this patient?

Collateral circulation > The lung is supplied by *dual circulation* from the pulmonary and bronchial system (collateral circulation). As a result, *lung infarction* only rarely develops as a complication of *pulmonary embolism* (PE) > When a clot occludes the pulmonary artery, the bronchial artery can continue to supply nutrients to the tissue.

uWORLD: A community hospital is experiencing an increased incidence of nosocomial pneumonias. The microorganism isolated in many cases is best visualized with silver stains and demonstrates slow growth on buffered charcoal yeast extract medium (supplemented with L-cystein and iron). Which of the following is the most likely cause of the increase incidence of nosocomial pneumonia's in this hospital?

Colonization of the hospital water system > Legionella pneumophila commonly contaminates natural bodies of water, municipal water supplies, and water-based cooling systems. The organism is inhaled in aerosolized water and establishes infection via the pulmonary route. Culture must be performed on buffered charcoal yeast extract agar summplmented with L-system

___________________ is an encapsulated yeast that causes meningoencephalitis and pulmonary cryptococcosis primarily in immunocompromised patients, particularly those with HIV infection. However, it is found extracellular as its large, polysaccharide capsule inhibits phagocytosis by macrophages.

Cryptococcus neoformans

uWORLD: A 35 yo man who received a kidney transplant one year ago comes to the ED with pleuritic chest pain and cough. He is found to have a low-grade fever. Chest x-ray reveals an infiltrate in the lower lobe of the right lung. The patient is started on broad-spectrum antibiotics, including vancomycin, ceftriaxone, and azithromycin. Despite these measure, his condition worsens over the next several days. Bronchoscopy with bronchoalveolor lavage is performed. Mucicarmine staining of his bornchoalveloar fluid reveals red staining. What is the most likely cause of this patient's condition?

Cryptococcus neoformans > Cryptococcus neoformans is a yeast that has an antiphagocytic polysaccharide capsule, a major virulence factor. Methenamine silver stain can identify the yeast form of Cryptococcus, seen in tissue as round cell with narrow-based buds. The polysaccharide *capsule* appears as a clear, unstained zone with *india ink* and stains *red* with mucicarmine.

uWORLD: A 60 yo woman come to the ED with SOB and productive cough. She has a 30 pack year smoking history. The patient's oxygen saturation is 88% on room air. On physical examination, she appears uncomfortable and uses her accessory respiratory muscles. Expiratory wheezes are heard throughout the lungs. The patient is started on high-flow oxygen supplementation. Shortly afterward, she becomes increasingly lethargic and confused. This patient's clinical decline is most likely attributable to an increase in which of the following?

Dead space ventilation > Administration of excessively high oxygen concentrations in patients with chronic obstructive pulmonary debases can lead to increased CO2 retention (Oxygen-induced hypercapnia), resulting in confusion and depressed consciousness. This is due in part to the reversal of hypoxic pulmonary vasoconstriction, which increases physiologic dead space as the poorly ventilated alveoli are perused (Ventilation-perfusion mismatch) > Hypoxia normally causes vasoconstriction of the pulmonary arterioles, which suits blood toward alveoli with the highest oxygen content, thereby minimizing physiologic dead space. Hyperopia reverses pulmonary vasoconstriction, *increasing physiologic dead space*, as poorly ventilated alveoli and perfused (*ventilation-perfusion mismatch*)

uWORLD: A 32 year old female with claustrophobia (fear of closed spaces) is stuck in a malfuncitoning elevator. She experiences severe anxiety as well as dizziness, weakness and blurred vision. Which of he following is most likely cause of this patient's symptoms?

Decreased arterial partial CO2 tension > The patient in this vignette is having a panic attack. Panic attacks are typically accompanied by hyperventilation and decreased pCO2 (hypocapnia). Hypocapnia can cause decreased cerebral perfusion and thus neurological symptoms > Hypcapnia causes cerebral vasoconstriction and decreased cerebral blow food. .

uWORLD: A 4 yo boy is brought to the ED for difficulty breathing and a productive cough. He has a past medical history of recurrent episodes of sinusitis and otitis media. The respiratory infections typically have a protract course but response to high-dose antlobtic therapy. Pulmonary auscultation shows crackles and wheezing. Cardiovascular examination shows a cardiac point of maximal implies that is palpated at the right 5th intercostal space. What its most likely to be dysfunctional in this patient?

Dynein arms > The patient's history of recurrent respiratory infections and dextrocardia on the physical examination are suggest of *Kartagener syndrome* a form of *primary ciliary dyskinesia (PCD)*. This autosomal recessive condition is caused by a wide variety of mutations that impair ciliary structure of function. > Primary ciliary dyskinesia can result form failure of the *dynein arms* to develop normally. > patients with PCD experience *recurrent respiratory infections* (eg, chronic sinusitis, bronchiectasis) as a result of impaired mucocillary clearance. In addition, impaired ciliary moment during embyogensis can cause *situs inversus* (reversed right/left positioning of internal organs). *Infertility* in men (impaired sperm motility( and women (immobility of fallopian tube cilla) is another frequent manifestation. > Karagener syndrome is a form of primary ciliary dyskinesia characterized by the triad of situs inverses, chronic sinusitis, and bronchiectasis. it occurs due to mutations that impairs the structure or function of cilia. Cystic fibrosis also causes chronic respiratory infection, but it is not associated with situs inverses.

uWORLD: A viral particle isolated from the nasal exudate of a 10 yo male are shown to lose their infectivity once exposed to ether. Ic can be concluded that the viral particles are most likely: (Heat-stable, defective, enveloped, icosahedral, recombinant)

Enveloped > Ether and other organic solvents can inactivate the "enveloped" viruses, which by definition have an outer lipid bilayer coat acquired from the host cell plasma or nuclear membranes. > Loss of infectivity after ether exposure is an characteristic feature of enveloped viruses. > Non-eveloped viruses are generally resistant to the action of ether.

uWORLD; A 53 yo man comes to the ED due to progressively worsening SOB, nonproductive cough and low-grade fevers over the past 2 weeks. He has not had a runny nose or sore throat and does not recall any sick contacts. He received a lung transplant for idiopathic pulmonary fibrosis 4 months ago. His medications include immunosuppressants and trimethoprim-sulfamethoxazole. His temperature is 100 F. Chest x-ray reveals diffuse interstitial infiltrates bilaterally. A decrease in pulmonary function is noted on testing. Which characteristics the organism most likely responsible for this patient's current condition?

Enveloped double-stranded DNA virus > Transplant patients are at risk of a variety of unusual infections due to their immunocompromised state. *Cytomegalovirus (CMV)* is particularly common in patients with lung *transplants* (typically occurring within the first few months after transplant). Most but not all transplant centers practice universal prophylaxis for lung transplant recipients (eg. valganciclovir). > CMV is an *enveloped double-stranded DNA* virus belonging to the Herpesvirdiae family. > Histopathology of lung biopsy from patients with CMV *pneumonitis* (most common form of tissue-invasive CMV following lung transplant) may show enlarged cells with *intranuclear and intracytoplasmic inclusions* (viral particles); there is often a surrounding halo (*owl's eye*)

uWORLD: An infant is evaluated at the office after developing prominent oral thrush, interstitial pneumonia, and severe lymphopenia during the first year of life. The patent was born full-term to a 25 yo woman who received no prenatal care and is an intravenous drug abuser. She breastfed her son for the first mouth but then switched to formula feedings. The mother is currently unwilling to undergo any kind of testing. She tells the heath care provider, "Just take care of my son". Which of the following prenatal interventions would have most likely prevented this infant's condition? (Enzyme inhibitor drug, interferon therapy, killed vaccine, live attenuated vaccine, prenatal vitamins, toxoid vaccine, viral component vaccine)

Enzyme inhibitor drug > Oral thrush, interstitial pneumonia, and severe lypmhopenia during the first year of life are consistent with mother-to-child *vertical transmission of HIV-1*. The risk of HIV infection occurring in an infant born to an HIV-postive mother who received no prenatal *antiretroviral therapy* (ART) can be as high as 35%. ART during pregnancy reduces the risk of perinatal transmission to 1%-2%. All pregnant women with HIV should take ART, regardless of their CD4 cell count or viral load.

uWORLD: A 56 yo male smoker presents to your office complaining of a chronic rough. He has been hospitalized several times in the past for respiratory infections requiring parental antibiotics. he has smoke cigarettes fro the past 30 years despite multiple attempts to quite. Today, his oxygen starution is 90@ at rest and decreases to 84% with moderate exertion. Which of the following finding is most likely in this patient?

Increased erythropoietin production > COPD can cause hypoxia sufficient to stimulate increased erythropoietin production by the cortical cells of the kidney. NOTE > In COPD, the FRC is increased, but it is decreased in restrictive lung disease.

uWORLD: A 33 yo man comes to the office due to worsening exertion dyspnea. He states that in the past he was a frequent jogger, but now he becomes short of breath while walking his dog in the park. He has no other significant past medical history. The patient smokes a pack of cigarettes daily. BMI is 27. Cardiac examination is unremarkable. Laboratory analysis reveals markedly decreased serum alpha-1 antitrypsin levels. Which of the following pulmonary function test patterns is most likely to be present in this patient? FEV1/FVC? TLC? DLCO?

FEV1/FVC = Decrease TLC = Increase DLCO = Decrease > Emphysema most commonly results from chronic smoking but can also occur in genetically predisposed individuals with alpha-1 antitrypsin deficiency.

The Ried index is the ratio of the thickness of the _____________ in the bronchial wall submuscoas to the thickness of the bronchial wall between the respiratory epithelium and bronchial cartilage.

Mucous gland layer

A 32 yo man is hospitalized following a motor vehicle accident during which he sustained bilateral femur fractures, pelvic fracture, and urethral injury. The patient has no other intraabdominal or chest injury. On the third day of his hospitalization, he reports shortness of breath. The patient's temperature 98.8 F, heart rate is 110 and regular, respirations are 20, and oxygen saturation is 84% on room air. He appears confused and agitated. Physical examination shows a petechial rash on his chest. ECG shows sinus tachycardia with no ischemic changes. Which of the following histologic changes has most likely taken place in this patient lung tissue?

Fat micro globules in pulmonary arterioles > Fat embolism syndrome should be strongly suspected in a patient with severe long bone and/or pelvic fractures who develops acute-onset neurologic abnormalities, hypoxemia, and a petechial rash. Occlusion of the pulmonary micro vessels by fat globules is an early histologic finding of this syndrome.

uWORLD. A 56 yo African American woman with a history of hypertension is brought to the ED because of three days of pyuria and back pain. On physical examination, she has a temperature of 102.6 F, a blood pressure of 70/40, a pulse of 130, and respirations of 28. She is confused and her skin is diffusely warm to the touch. She has suprapubic and costoverebral angle tenderness, but no rashes or edema. Her complete blood count demonstrates leukocytosis with increased neutrophils, but is otherwise normal. Coagulation studies are normal. Her urinalysis is positive for leukocyte esterase and nitrites and has numerous bacteria. Several hours later she becomes increasingly hypoxic requiring mechanical ventilation. Her respiratory symptoms are most likely due to which of the following pathology conditions?

Fluid accumulation in the alveolar spaces > This patient with fevers, hemodynamic instability, tachypnea, and evidence of a urinary tract infection has urospesis which is likely complicated by *acute respiratory distress syndrome (ARDS). *Sepsis* and pulmonary infection are the two most common *risk factors* for ARDS. > During sepsis, cytokines such as tumor necrosis factor, interleukin (IL)-1, IL-6, and IL-8 begin to circulate in response to the infection, causing activation of the pulmonary epithelium. This results in increased recruitment and extravasation of neutrophils into the lung tissues provoking an inflammatory response that leads to capillary damage and the leakage of protein and fluid into the alveolar space. > As more alveolar space becomes *fluid-filled* patients develop worsening hypoxia and respiratory failure. More than half the cases of ARDS present within 24 hours of the inciting risk factor (eg. pneumonia, sepsis, trauma)

uWORLD: A 32 yo woman comes to the ED with severe SOB. She has no chest pain and does not use tobacco. Her temperature is 98 F, blood pressure is 132/81, pulse is 102, and respirations are 30. During physical examination, her condition deteriorates and she is emergently intubated due to acute respiratory failure. Shortly after intubation, she goes into cardiac arrest and dies despite resuscitation efforts. Review of the patient's medical records show that she had been prescribed multiple medications but had a history of noncompliance. An autospy is performed and shows hyper inflated lungs, airway mucus plugging, and cellular infiltration of the bronchial wall. Long-term use of which of the following drugs would have best prevented the cellular reaction in this patient's airways? (Albuterol, fluticasone, ipratropium, magnesium sulfate, montelukast, theophylline)

Fluticasone > Corticosteroids have the strongest and most predictable effects on the inflammatory component of asthma. Inhaled corticosteroids form the cornerstone of chronic therapy for patients with persistent asthma and can reduce the number of severity of acute asthma exacerbations. > *Glucocorticoids* have pronounced anti-inflammatory effects on the respiratory epithelium and are used for both chronic asthma management (inhaled steroids) and during acute exacerbation (systemic steroids). They inhabit the formation of inflammatory mediators (eg, cytokines, prostaglandins, leukotrienes) implicated in bronchial asthma. > They also reduce leukocyte extravasation and induce apoptosis of inflammatory cells (eg. macrophages, lymphocytes, and eosinophils). As a result, patients treated with glucocorticoids for a few weeks show significantly *reduced airway inflammation*. > In addition, glucocorticoids decrease the amount of mucus produced by goblet cells, further reducing the airway obstruction.

uWORLD: A 65 yo male dies in the hospital of an overwhelming pulmonary infection. Autospy shows a right lower lung lobe that is pale, firm, and airless. Histologically, the alveoli are filled with exudate containing neutrophils, fibrin and some fragmented RBCs. The patient died during which phase of his disease?

Gray hepatization > In Gray hepatization, RBCs disintegrate

A 43 yo man brought to the ED has the following arterial blood gas reading: Low: pH High: pCO2 Low: HCO3- The patent most likely suffers from which of the following conditions?

Heroin overdose > Hypoventilation causes an increase in arterial pCO2 and a decrease in the serum pH (respiratory acidosis). Acute respiratory acidosis presents with a low pH, a high pCO2 and a normal to mildly increased HCO3- because renal compensation requires a least 24 hours of persistent respiratory acidosis. Chronic respiratory acidosis is characterized by a low normal normal pH, a high pCO2, and a high HCO3- (>30).

uWORLD: A 54 y/o man diagnosed with HIV infection 2 years ago develops a cough, low-grade fever, and hepatosplenomegaly. He has lost 3.6 kg (8 lb) over the past month. Histology findings show small ovoid bodies within a macrophage. What fungi is most likely the cause of this patient's condition

Histoplasma capsulatum > Histoplasma capsulatum can survive intracellular within macrophages. It causes a disseminated mycosis in immunocompromised patients. The clinical feature include systemc syndromes, (fever and weight loss), painful oral ulcers, lymphadenopathy, and hepatosplenamegaly.

uWORLD: A 42 yo woman comes to the office due to several months of episodic SOB and cough. The patient has a history of migraines and occasionally takes acetaminophen. She is a school teacher and does not use tobacco, alcohol, or illicit drugs. Her temperature is 97 F, blood pressure is 110/80 and pulse is 84/min. Pulse ox show 96 on room air. Lung examination and chest x-ray are normal. Spirometry shows FEV1 is 71% of the predicted value. Sputum analysis shows predominant eosinophils. This patient's symptoms are most likely related to exposure to which of the following?

House dust mites > This patient's *intermittent respiratory symptoms* (eg, dyspnea, cough) in the setting of a normal chest x-ray, sputum eosinophils, and reduced FEV1 (<80% predicted) are highly suggestive of *asthma*. Common asthma triggers include exercise, cold air, respiratory infection, and exposure to inhaled allergens (Eg. dust mites, cockroaches, pet dander, mold, pollen).

TH2 type CD4+ helper T cells predominately drive ___________________ immune responses. Their products include IL-4, which promotes IgE antibody production by B-cells, and IL-5 which promotes the production and activation of eosinophils and B-cell synthesis of IgA.

Humoral

uWORLD: A 24 yo Caucasian male is admitted to the hospital with a 1 week history of fever, cough, and chest pain. Lung auscultation reveals crackles over the right lower lung lobe. When placed in an upright glass tube, his anti coagulated RBCs fall at a rate of 35 mm per hour. This finding is most likely related to the action of which of he following substances? (Bradykinin, LTB4, Erythropoietin, IL-6, Platelet activating factor, Thromboxane A2)

IL-6 > When an injury occurs, local neutrophils and macrophages release tumor necrosis factor-alpha (TNF-alpha), IL-1, and IL-6 into circulation. These particular cytokines mediate systemic inflammatory response, in part by stimulating hepatic production of acute phase reactants (eg, fibrinogen, ferritin, C-reactive protein, serum amyloid A, serum amyloid P, complement factors). > Many acute-phase proteins bind to microbes and fix complement. When increased, the acute-phase protein fibrinogen causes erythrocytes to form stacks (rouleaux) that sediment at a faster rate than do individual erythrocytes. > This aggregation rate is termed the erythrocyte sedimentation rate, or ESR, and non-specific marker of inflammation.

uWORLD: A 52 yo homeless man is found unresponsive on the street and brought to he ED. The patient's past medical history is unknown. His temperature is 97.2 F and blood pressure is 108/62. He is unresponsive to cereal and tactile stimuli. An arterial blood gas analysis shows that the partial pressure of oxygen in his arterial blood is 60 mmHG. The partial pressure of oxygen in his alveoli is calculated to be 71 mmHg. With of the following is the most likely cause of this patient's symptoms? (Diffusion impairment, hypoventilation, left-to-right shunt, right to left shunt, ventlaiton/perfusion mismatch)

Hypoventilation > The discrepancy between alveolar and arterial O2 concentraiotn is termed the alveolar-arterial (A-a) gradient. In healthy individuals, the A-a gradient is normally between 5-15 mm Hg, with older individuals having a higher A-a gradient due to an age-related decrease in O2 diffusing capacity. > This patient presents with low PaO2 and PAO2 and a *normal A-a gradient (71-60 =11). This indicates that his low PaO2 is directly due to his low PAO2 and not caused by ventilation-perfusion (V/A) mismatch or O2 diffusion impairment. Possible causes of hypoxemia in the setting of a normal A-a gradient includes *alveolar hypoventilation* and inspiration of air at high altitudes. > Hypoventilation is common in patients with surpassed central respiratory drive (eg, sedative overdose, sleep apnea) or those with diseases that decrease inspiratory capacity (eg. myasthenia gravis, obesity) NOTE > V/Q mismatch is a common cause of hypoxemia. poor ventilation of a lung segment with normal perfusion (eg, pneumonia, obstructive pulmonary disease) resulting in physiologic shunting, leading to an increased A-a gradient.

uWORLD: Sensitized TH2 cells exposed to inhaled allergens can produce as substance that promotes B-lypmohte class-switching to synthesize IgE. This substance is:

IL-4

uWORLD: A 63 yo man comes to the office for evaluation of 3 months of increasing cough with occasional hemoptysis, night sweats, and unintentional weigh loss. He recently migrated from South Africa, where he worked in the gold mines. The patient has a history of silicosis diagnosed 10 years ago. Vital signs show a low-grade fever. Exmianaiton reveals diffuse, fine crackles with right upper lobe predominance. Chest x-ray shows diffuse small nodules, hilar adenopathy with prominent calcifications, and a right upper lobe cavitary lesion. Sputum is sent for appropriate staining and culture. This patient's increased susceptibiliy to the current infection is best explained by which of the following factors?

Impaired macrophage function > *Silicosis* is distinguished by clacficaiton of the rim of hilar nodes (Eggshell calcification) and birefringent silica particles surrounded by fibrous tissue on histology. It has long been associated with increased risk of *tuberculosis* > Silicosis impairs the *macrophage* effector arm of cell-mediated immunity, which is integral to the immune response against mycobacteria. It is thought that macrophage phagolysosomes are disrupted by internalized silica particles, causing release of the particles and viable mycobacteria, which inhibits the immune system's ability to control the infection. This extracellular release of lysosomal enzymes is thought to contribute to alveolar and interstitial lung injury in silicosis. Prolonged exposure of macrophage to silica particles also appears to increase macrophage apoptosis.

Decreased chest wall compliance would increase the work of breathing, potentially resulting in respiratory muscle fatigue and alveolar hypoventilation. This would cause an ______________ in PaCO2 levels.

Increase

Respiratory muscle fatigue, poor respiratory drive, or upper airway obstruction would impair alveolar ventilation, resulting in __________________ PaCO2 and decreased PaO2

Increased

uWORLD: A 2 day old boy was brought to the intensive care unit from the nursery with tachypnea and hypoxia. the patient was born at 33 weeks, and his mother did not receive prenatal care. Physical examination shows cyanosis. He was intubated but died 2 days later. An autospy is performed, and a sample of the patient's lung tissue is obtained. Analysis of several large, cuboidal alveolar cell under electron microscopy reveals descried number of granules containing parallel stacks of membrane lamellae. Which pathological processes is most likely to result from the abnormal autospy finding?

Increased alveolar tendency to collapse > When there is insufficient surfactant, as in neonatal respiratory distress syndrome, the result is collapse of alveoli (*atelectasis*) due to increased surface tension. > Surfactant is stored and transported to the cell surface by *lamellar bodies* (organelles containing parallel stacks of membrane lamellae).

uWORLD: A 71 yo man with COPD comes to the ED with severe SOB and wheezing. HE is admitted to the hostpial for COPD exacerbation and started on bronchodilators and systemic corticosteriods. His symptoms improve over the next 2 days. Before being discharged, his nurse administers a vaccine containing inactivated viral components that he normally receives every year. He is subsequently exposed to the liv virus through natural infection. The patin's prior exposure to the vaccine is most likely to result in which?

Inhibition of viral entry into cells > Inactive (killed or component) viral vaccines predominately generate a humoral immune response instead of a cell-mediated immune response. In contrast, live-attenuated viral vaccines can generate a strong cell-mediated immune response in addition to providing humoral immunity.

______________ is used for the treatment of hairy cell leukemia, condyloma acuminate, hepatitis B virus infection, and Kaposi sarcoma.

Interferon-alpha

uWORLD: A 5 yo boy with severe recurrent respiratory infection is undergoing evaluation. It is determine that a specific type of bacteria cause his infection. Further testing reveals that the patient's T-lymphocte are found to lack the IL-12. Supplement with which substance would improve the patient's condition?

Interferon-y (INF-y) > IL-12 simulates the differentiation of "naive" T-helper cells in the Th1 subptopution. Patients with IL-12 receptors definecy suffers from severe mycobacterial infections due to the inability to mount a strong cell-mediated granulomatous immune response. They are treated with IFN-y

uWORLD a 2 day-old newborn develops leathery and respiratory distress. Blood cultures grow beta-hemolytic Gram-postive coco in chain that are bacitracin-resistant. Which of the following measures could have prevented this patient's condition most effectively?

Intrapartum ampicillin > The finding of Gram-postive cocci in chains indicates Streptococcus as Staphylococci classically form cluster. Group A streptococci and Group B streptococci are beta-hemolytic, but bacitracin resistance excludes S. pyrogenese (GAS) and indicates S. agalasctiae (GBS) > Universal prenatal screening for group B strep colonization by vaginal-rectal culture at 35-37 weeks gestation is recommended to identify colonized women who require INTRAPARTUM antibiotics, most frequently with penicillin or ampicillin, to prevent neonatal GBS sepsis, pneumonia and meningitis.

uWORLD: A 62 yo man comes to the physician because of recent weight loss, cough, and occasional hemoptysis. His past medical history is significant for poorly controlled diabetes mellitus and chronic obstructive pulmonary treated with bronchodilators and oral corticosteroids. Chest x-ray shows pulmonary infiltrates and an area of cavitation in the right upper lobe. Sputum microscopy shows acid-fast bacilli. Which of the following is the most accurate statement concerning this patient's pulmonary infection?

It originated from reactivation of an old infection > Primary tuberculosis causes the formation of Ghon foci in the lower lungs. Secondary (Reactivation) tuberculosis occurs in patients with prior tuberculosis infection that never cleared completely. Reactivation tuberculosis occurs most often in immunosuppressed patients and is characterized by apical cavitary lesions and hemoptysis.

uWORLD: A 65 yo man comes to the physician due to fevers, chills, chest pain, and a productive cough for the last 2 days. He drinks 6-10 beers daily an has a 45 pack year smoking history. His temperature is 104.4 F, blood pressure is 100/70, pulse is 95, and respirations are 28. On examinations, he is in mild respiratory distress. There are crackles and decreased breath sounds in the right upper lobe. His chest x-ray shows a large right upper lobe infiltrate. Sputum examination shows encapsulated gram-negative bacilli. Sputum culture grows pink-colored mucoid colonies on MacConkey agar. What organism is most likely the cause of this patient's condition? (Chlamydia, Klebsiella, Legionella, Mycoplasma, Pseudomonas, Stap aureus, Streph pneumoniae)

Klebsiella pneumoniae > Klebsiella is an encapsulated, lactose-fermenting, gram-negative bacillus the appears mucoid in culture. It causes pneumonia in subjects with impaired host defenses, espiscally alcoholics. Klebsiella pneumonia is characterized by tissue necrosis and early abscess formation with production of thick, mucoid, blood-tinged sputum (currant jelly sputum)

_____________________ is produced by *Clostridium perfringens* and hydrolyzes lecithin in cell membrane. This results in cell lysis and the development of gas gangrene.

Lecithinase (toxin A)

_______________ is an occasional cause of septicemia and purulent meningitis in neonates

Listeria monocytogenes

uWORLD: A 56 yo woman is brought to the ED due to a 2 day history of high fever, headache, mild confusion, and dry cough. She also has mild abdominal discomfort and watery diarrhea. The patient recently returned from a cruise to Hawaii. Her other medical problems include HTN and hyperlipidemia. She has smoked 1 pack of cigarettes daily for over 20 years. Her temperature is 104 F, blood pressure is 104/63, pulse is 85 and respirations are 24. Lung examinations reveals lower lobe crackles with no wheezing. Her abdomen is soft, non-distended, and non-tender. Chest x-ray shows bilateral lower lobe interstitial infiltrates. Which of the following additional findings is most likely to be present in this patient?

Low serum sodium > *Legionelleosis* can be dived into Pontiac fever (an acute, flu-like, self-limned disease) and the more Legionnarie's disease. > Legionnaires' dises should be suspected in patients with recent exposure to *contaminedated water* (sporadic cases or common-source outbreaks in cruise ships, spas, hospitals, or air-conditioned hotels), radiographic evidence of pneumonia (typically patchy infiltrates that may progress to consolidation), *high fever, sometimes associated with *relatve bradycardia, neurologic* symptoms (eg, confusion, headache), and *gastrointestinal (GI) symptoms (eg diarrhea). > The most common laboratory abnormality is *hyponatremia*, frequently associated with Legionella but not other causes of pneumonia. > The hyponatremia may be related to inappropriate ADH secretion and/or renal tubulointerstital disease impairing sodium reabsorption (possibly due to either direct effect of Legionella, cytokines, or natirjuertic peptides)

_________________ is found in the cell wall of *Streptococcus* species. It binds to the factor H to prevent opsonization and destruction of streptococci by the alternative complement pathway.

M protein

____________________ inhibits calcium influx into airway smooth muscle and promotes bronchodilation when given intravenously during an acute asthma flare-up. It also reduces inflammation by stabilizing T cells and inhibiting mast cell degranulation; however, it is less effective than glucocorticoids.

Magnesium sulfate NOTE > Leukotrienes are potent inducers of airway bronchoconstricion and inflammation. Leukotriene inhibitors (eg. montelukast) are useful for long-term control in some patients with asthma. However, they possess less efficacy and have fewer anti-inflammatory properties than inhaled glucocorticoids. > Theophylline, a phosphodiesterase inhibitor, results in increased intracellular cyclic AMP, which contributes to its role as a bronchodilator. Theophylline also has anti-inflammatory effects, although they are far less potent than glucocorticoids.

uWORLD: A 64 yo man comes to the ED with a 2 day history of fever, chills, and productive cough. His temperature is 102 F, BP is 108/52 and pulse is 102, and respirations are 26/min. Crackles and bronchial breath sounds are heard over the right lower lung. There is dullness to percussion over the same area. Chest x-ray reveals right lower lobe consolidation and a right-sided pleural effusion. The patient is started on the appropriate treatment, and a diagnostic thoracentesis is performed that shows an uncomplicated parapneumonic effusion. When a sterile sample of the inflammatory exudate is experimentally introduced into normal human tissue, rapid neutrophil locomotion is observed. Which of the following components of the exudate is most likely responsible for this observed effect? (Bradykinin, C4a, IFN-y, IL-4, Leukotriene B4, Thromboxane A2)

Leukotriene B4 > *Parapneumonic effusions* occur frequently in bacterial pneumonia as a result of *excudative* fluid accumulation within the pleural space. Infections and other forms of inflammatory tissue injury cause increased vascular permeability, leading to the formation of protein-rich exudates that contain a variety of biologically active substances. Under the influence of inflammatory stimuli, cell membrane phospholipids release arachidonic acid, a precursor to the eicosanoid inflammatory mediators (eg, prostanoids, leukotrienes, lipoxins). The most potent chemotactic eicosanoid is *leukotriene B4*. > In contract, the cysteineyl-containing leukotrienes (eg. LTC4, LTD4, LTE4) cause bronchospasm and increase bronchial muscus secretions and are important in asthma pathogenesis. NOTE: > Complement componenets C3a, C4a, and C5a are inflammatory anaphylotoxins that trigger histamine release form mast cells, resulting in vasodilation and enhanced vascular permeability. C5a also recruits and activates neutrophils, monocytes, eosinophils, and basophils. C4a is the least active of these compliment components and plays a minor role in leukocyte recruitment.

uWORLD: A 56 yo smoker with a persistent dry cough comes to the physician due to recent-onset headaches and dyspnea. He also complains of having a "puffy face" for 2 weeks but denies any other medical problems. He has no shoulder pain. Physical examination shows symmetrical facial swelling and conjunctival edema. His pupils are equal, round, and reactive to light. Dilated vessels are seen over this neck and upper trunk. Heart sounds are clear. This patient's condition is most likely caused by which of the following?

Mediastinal mass > Intrathroacic spread of bronchogenic carcinoma may lead to compression of the superior vena cava, causing impaired venous return form the upper part of the body. Signs and symptoms include dyspnea, facial swelling, and halted collateral veins in the upper trunk. > This patient's symptoms are consistent with impaired venous return from the upper body. The superior vena cava provides venous drainage from the head, neck, upper trunk, and upper extremities. It is formed by the union of the right and left brachiocephalic veins behind the 1st costal cartilage on the right. It has thin walls and is easily compressed by mediastinal masses. > Compression of the superior cava causes a combination of symptoms called *superior vena cava syndrome*. Affected patients complain of dyspnea, cough, and *welling of the face*, neck, and upper extremities. Headaches, dizziness, and confusion may occur due to cerebral edema and elevated intracranial pressure. > Dilated collateral veins may be seen in the upper torso. Lung cancer, followed by non-Hodgkin lymphoma, is the most common cause of superior vena cava dyrnome.

uWORLD: A 45 yo man comes to the ED due to 2 weeks of chest pain and cough. He has a history of advanced HIV and has taken his antiretroviral medications inconsistently over the past few months. Temperature is 100.6 F. Crackles are heard on pulmonary examination. CD4 cell count is 98/mm. Chest x-ray reveals nodules and hilar lymphadenopathy. A bronchoscopy is performed. Mucicarmine staining of the patient's bronchoalveolar fluid shows budding yeast forms with thick capsules. Symptomatic infection with the organism causing this patient's condition most common manifests as which of the following?

Meningoencephalitis > Budding yeasts with thick capsules are chracerstic of Cryptococus neoformans, which typically affects only *immunocompromised* patients (*opportunistic pathogen*). C neoformans is present in soil and pigeon droppings; it is transmitted by the respiratory route but not acquired via person-to-person contact. > In individuals with an impaired cellular immune response, C neoformans can cause symptomatic disease, most commonly *meningoencephalitis*

*Minute ventilation* is the total volume of new air that enters the respiratory pathways per minutes. it can be calculated using the following equation: ____________________ Alveolar ventilation refers only to the volume of new air reaching the gas exchange areas per minute (i.e., reaching the alveoli and respiratory bronchioles). Therefore, alveolar ventilation does not include dead space volume. It can be calculated using the following formula __________________

Minute ventilation (L/min) = tidal volume (L) X breaths/min Alveolar ventilation (L/min) = (tidal volume - dead space volume) X breaths/min

uWORLD: A 23 yo male is being evaluated for persistent cough and a pulmonary infiltrate detected on chest X-ray. 3 mL of this patient's blood is added to an anticoagulated tube and placed into iced water. Several minutes later clumping is detected inside the tube, but it rapidly uncoagulates when the tube is warmed while being held it in the observer's hand. Which of the following organisms is most likely responsible for this patient's condition?

Mycoplasma pneumonia > The question stem is describing the effect of cold agglutinins. As their name implies, cold agglutinins cause the agglutination or clumping of RBCs when a sample of blood containing cold agglutinins is chilled. > Cold agglutinins are antibodies that are produced in response to Mycoplasma pneumoniae infection. These antibodies are directed against antigen in the cell membrane of M. pneumoniae that happen to be homologous to antigens that are present on the surface of human erythrocytes. > The cold agglutinins are responsible for the transient anemia that can be documented in many patients with M. pneumonia infection. > Cold agglutinins are also associated with Epstein-Barr virus infection and hematologic malignancies in addition to infection with Mycoplasma pneumoniae.

uWORLD: Health authorities are investigating an outbreak of respiratory infections among a group of military recruits. Fifteen recruits reported persistent cough, low-grade fever, and malaise. Apart from the low-grade fever, physical examination was largely unremarkable. Chest x-rays were all notable for pulmonary infiltrates that appeared more severe than what would have been expected based on assessment of the patient's clinical status. Sputum specimens were obtained, and the causative organism required a complex acellular medium enriched with cholesterol to grow. Which of the following organism is the most likely cause of the outbreak?

Mycoplasma pneumoniae. > Mycoplasma pneumoniae is the causative agent of "walking pneumonia" an infection typically characterized by a nagging nonproductive cough, low-grade fever, and malaise. Often, the chest x-ray suggests a severe pneumonia even though the patent appears relatively well. > *Mycoplasma species require cholesterol supplementation* to grow on artificial media.

uWORLD: A 44 yo man comes to the ED with a 3-day history of fever, chills, malaise, dyspnea, and a cough productive of "greenish" sputum. The patient has no prior medical problems and has never been hospitalized. He has a 25-pack-year smoking history and drinks 4-5 beers a week. His temperature is 103 F, blood pressure is 130/80, pulse is 98, and respiration are 20. On examination, dullness to percussion, crackles, and egophony are present at the right lung base. The remainder of the examination is normal. Chest x-ray shows a dense infiltrate occupying the entire right lower lobe. Which of the following most likely accounts for the color of this patient's sputum? (Epithelial necrosis, hemolysis, high bacterial load, mucopolysaccharides, myeloperoxidase)

Myeloperoxidase > The middle-age man presenting with new-onset fevers, productive cough, and a dense lobar infiltrate likely has community-acquired *pneumonia* (CAP). CAP in otherwise healthy individuals is most commonly caused by Streptococcus pneumoniae, the most common bacterial etiology worldwide. Tobacco use further increases its risk. In non elderly patients, pneumococcal pneumonia presents with abrupt-onset fevers, rigors, tachypnea, and productive cough with *consolidation* in one lobe of the lungs > Neutrophil *myeloperoxidase* is responsible for the *green* color of pus and sputum in bacterial infections. It is a blue-green heme-based pigmented molecule contained with the zoophilic granules of neutrophils and catalyzes the production of hypochlorous acid from chloride and hydrogen peroxide fudging the phagocytic respiratory burst.

uWORLD: A 4 yo has just recovered from severe staphylococcal pneumonia. He has a history of recurrent lymphadenitis and skin infections. Dihydrohodamine flow cytometry testing reveals an absence of the green fluorescence that is characteristic of normal neutrophils. This patient's condition is most likely due to impaired activity of which enzyme?

NADPH oxidase > Chronic granulomatous disease (CGD) is most commonly due to an X-linked mutation affecting NADPH oxidase. Deficiency of this enzyme leads to an inability of neutrophils to form the oxidative burst necessary to kill organisms in their phagolysosomes. CGD can be diagnosed by absence of the normal blue and fluorescent green pigment produced by ntiroblue tetrazolium test and dihydrohodamine flow cytometry test, respectively.

uWORLD: A male neonate is born prematurely to 22 yo primigravida and experiences severe respiratory distress. The distress. The distress resolves with treatment, and by two weeks age, the patient has adequate respiratory function. Ophthalmoscopy later shows abnormal retinal vascularization that extends into the vitreous. the retinal findings in this patient are most likely related to:

Neonatal oxygen suppementation Respiratory distress in a premature neonate is most commonly due to pulmonary surfactant deficiency resulting in hyaline membrane disease. Treatment involves administration of supplemental oxygen at high concentrations, nasal continuous positive airway pressure, and/or technical ventilation with intratracheal surfactant. One potential adverse effect of oxygen therapy is retinal damage. > Temporary local hyperoxia in the retina is thought to induce changes that cause up-regulation of proangiogenic factors such as vascular endothelial growth factor (VEGF) upon return to room air ventilation. Retinal vessel proliferation (neovascularization) and possible retinal detachment with blindness may result. This complication of neonatal respiratory distress syndrome is referred to as retinopathy of prematurity or retrolental fibroplasia.

uWORLD: A 72 yo man is brought to the hostpial due to a week of progressive confusion and lethargy . He has also had persistent cough for the past several weeks and had 2 episodes of blood in the sputum. The patient has a history of hyperlipidemia, well-controlled HTN, and 48 pack-year of smoking. Lung auscultation reveals wheezing in the left lung. He is found to have hyponatremia with features suggestive of the syndrome of inappropriate antidiuretic hormone secretion. Chest x-ray reveals a mass in the upper lobe of the left lung. A bronchoscopic biopsy of the mass is performed, and the histopathology confirms an aggressive lung cancer. Which of the following is most likely to be seen in the biopsy sample of this patient?

Neural cell adhesion molecule > Small cell carcinomas frequently synthesize *hormones* or hormone-like substance (eg, vasopressin leading to syndrome of inappropriate antiderietic hormone secretion). Although small cell carcinoma is sensitive to chemotherapy and radiation, the 5 year survival rate of these patients is very low. > Small cell carcinomas show evidence of neuroendocrine differentiation. These tumors stain for *neuroendocrine markers*, such as *neural cell adhesion molecules* (NCAM, also known as CD56), neuron-specific *enolase, chromogranin*, and synaptophysin. Some small cell carcinomas express *neurofilaments*. Neurosecretory granules can be identified in the cytoplasm of the tumor cells on electron microscopy.

uWORLD: A 54 yo woman comes to the hospital due to a week of progressive fever and headache. She also had dyspnea, productive cough, fatigue, and night sweats. The patient underwent a renal transplant 2 years ago and is on immunosuppressive therapy. Her temperature is 101 F, blood pressure is 130/80, pulse is 90, respirations is 18. On examination, she is lethargic and has patch lung crackles with normal heart sounds and no focal neurological deficit. Her leukocyte count is 14,000. Chest x-ray reveals several nodules but no parenchymal infiltrate. Brain MRI shows a 1.2 cm ring-enchanting focal lesion with surrounding edema in the right frontal lobe. Sputum Gram stain is taken. What is the most likely the cause of the patient's condition?

Nocardia asteroides > Nocardia, weakly staining *gram-postive*, catalase-costive, rod-shaped bacteria found in soil and healthy gingiva. They form partially acid-fast, breaded *branching filaments* (resembling fungal hyphae but not as wide). > *Nocardiosis* typically affects the *lungs brain, or skin* and is mostly seen in *immunocompromised* patients. The *pulmonary form can result in cavitary infiltrates often initially misdiagnosed as tuberculosis; however, Mycobacterium tuberculosis would not appear as filaments of gram-positive organisms. > The other common presentation for nocardiosis is *brain abscess*. > The treatment of choice is trimethoprim-sulfamethoxazole.

uWORLD: A 42 yo man comes to his primary care physican due to daytime sleepiness. He often falls asleep during meetings and while watching television and has even fallen asleep while driving. The patient does not feel refreshed when waking and has occasional morning headaches. He has no had abnormal dreams or visual hallucinations when falling asleep or on walking. The patient has no significant past medical history and is a lifetime non-smoker. He drinks 2 or 3 beers on Friday nights. Blood pressure is 148/100, pulse is 78 and BMI is 32. Cardiopulmonary examination shows no abnormalities. Arterial blood gas analysis is normal. What is this patient's most likely diagnosis?

Obstructive sleep apnea > Obstructive sleep apnea is due to relaxation of oropharyngeal muscle one with occlusion of the upper airway. Symptoms include daytime sleepiness, *headaches*, and depression. > Complications include systemic and pulmonary hypertensions, right heart failure, and an increased risk for cardiac events. NOTE > Obesity hypoventilation syndrome by restricted expansion of the chest wall due to severe obesity. his leads to hypoventilation with a chronically elevated PCO2 and reduced PO2. > This patient's normal blood gases and mild obesity are not consistent with obesity hypoventilation.

uWORLD: A 35 yo man comes to the office due to worsening SOB. He is an avid hiker and began noticing this 18 months ago on challenging hikes; now even simple walks cause him to become short of breath. The patient has also noted mild wheezing and sputum production recently. He has no other medical problems and takes an antihistamine as needed without much improvement. the patient has smoked a half pack of cigarettes daily for the past 5 years. His father died from lung and liver disease at a young age. Spirometry sows reduced forced expiratory volume in one second and forced vital capacity. This patient's lower lung lobes are most likely to demonstrate which of the following findings.

Panacinar emphysema > His young age, family history of lung and liver disease, and minimal personal tobacco history are further suggestive of *alpha-1 antitrypsin (AAT) deficiency. > Deficiency can cause *panacinar emphysema*, which usually affects the *lower lung lobes* most severely (due to relatively greater perfusion, allowing a greater rate of neutrophil infiltration).

uWORLD: A 18 month old boy is brought to the physician by his parents for fever, runny nose, and sore throat. The physician reassures the parents and recommends supportive care with plenty of fluids. He sends them home with instructions to follow up if the boy's symptoms worsen. Two days later, the infant is brought to the ED with persistent fever, brassy cough, and difficulty breathing. Physical examination reveals stridor. Which of the following pathogens is most likely responsible for this patient's condition?

Paramyxovirus > Brassy, barking cough; dyspnea; and recent history of upper respiratory infection in a child are suggestive of trial larygnotrachetits (croup). The most common cause of croup is parainfluenza virus.

uWORLD: A 5 yo boy is brought to the ED by his parents for severe respiratory distress. He has not received any immunizations due to parental preference, but his past medical history is otherwise unremarkable. The patient appears very uncomfortable and anxious. He is leaning forward and making a loud harsh sound with every inspiration. His temperature is 103.4 F. The patient is intubated immediately. During intubation, the epiglottis is visualized and appears markedly swollen and erythematous. Which of the following is a major virulence factor of the pathogen causing this patient's infection?

Polyribosylribitol phosphate > The child's fever, dyspnea, stridor, and swollen epiglottis are consistent with *epiglottitis*, which is most commonly caused by *Haemophilus influenzae type b (Hib). The infected epiglottis swells and causes inspiratory stridor, dyspnea, and severe agitation. > The major virulence factor for Hib is its polysaccharide *capsule*, which is composed of the polymer *polyribosylribitol phosphate* (PRP). > The PRP capsule protects the bacterium against phagocytosis and complement-mediated lysis by binding *factor H*, a circulating complement control protein that prevents complement deposition on host cells.

uWORLD: A 48 yo male who died in a motor vehicle accident is found to have a small fibrotic focus in the lower lobe of the right lung and a calcified lymph node in the right lung hills. These autospy findings are most consistent with which of the following?

Primary exposure to M. tuberculosis > The finding of a fibrotic focus in the lower lobe of a lung in conjunction with a calf iced lymph node can occur in a variety of settings, with M tuberculosis infection one of the more likely. In M. tuberculosis infection, a lower lobe lung lesion (Ghon focus) accopamied by ipsilateral hilar adenopathy is described as a Ghon complex. The Ghon complex occurs during initial infection with M. tuberculosis.

uWORLD: A 58 yo man comes to the physician due to exertion dyspnea and cough. His symptoms started 6 months ago and have progressively worsened. His other medical problems include recurrent pyelonephritis, peptic ulcer disease, and rheumatoid arthritis. the patient had taken a medication for rheumatoid arthritis for many years, but stopped 1 year ago as the drug failed to improve his worsening hand arthritis. he does not use tobacco, alcohol, or illicit drugs. Physical examination shows joint diseases consistently with rheumatoid arthritis. Chest x-ray show diffuse pattern. What is the a likely explanation for this patient's pulmonary symptoms?

Pulmonary fibrosis > Pulmonary fibrosis presents with gradually progressive dyspnea and bilateral reticulonodular opacities on chest x-ray. Pulmonary function tests reveal a restrictive pattern and lung biopsy shows patch interstitial lymphocytic inflammation and fibrosis of the alveolar walls. > Rheumatoid arthritis can causes a variety of pulmonary manifestations; the most common is a form of interstitial lung disease similar to idiopathic interstitial pneumonia. > Methotrexate is a drug frequently used for rheumatoid arthritis treatment that can also cause interstitial pneumonitis and fibrosis.

uWORLD: A 54 yo man comes to the office due to daytime sleepiness and lack of energy. The symptoms began 6 months ago and have progressively worsened so that he feels "completely drained" by the end of the day. The patient's wife mentions that he snores loudly. His past medical history is unremarkable, although he has not been a physician in over 10 years. The patient does not use tobacco or alcohol, and he works in the warehouse of an agricultural supply company. BMI is 34. Physical examination shows a narrow oropharynx and a large neck circumference. The patient is at increased risk of developing which of the following? (Bronchiectasis, Hypertropic cardiomyoatphy, Laryngeal carcinoma, Narcolepsy, Pulmonary hypertension).

Pulmonary hypertension > Obstructive sleep apnea presents in obese individuals with excessive daytime sleepiness and signs of nocturnal upper airway obstruction (eg. snoring, gasping). The condition is associated with systemic hypertension. Prolonged, untreated obstructive sleep apnea can also cause pulmonary hypertension and right heart failure. > This patient, an obese man with loud snoring, daytime sleepiness, and suggestive examination findings (eg, thick neck, narrow airway) most likely has *obstructive sleep apnea* (OSA). > OSA is characterized by recurrent obstruction of the upper airway during sleep; each nocturnal episode of reduced ventilation causes transient *hypercapnia* and *hypoxemia*.

uWORLD: A 45 yo man comes to the urgent care clinic because of fever, severe headache, myalgia, and pleuritic chest pain. He has had these symptoms for several days. Physical examination shows fever and mild tachycardia. Lung auscultation reveals mid crackles. Radiographic examination is consistent with segmental pulmonary infiltrates. The patient fails to respond to empiric antibacterial antibiotic therapy. Microscopic examination of lung tissue obtained from this patient shows spherules packed with endospores. This patient' history is most likely to reveal which of the following?

Recent travel to Arizona > Cocidiocides immitis is a dimorphic fungus that has a mold form (hyphae) and a endosperm form (spherules containing endospores, a unique characteristic of Coccidiodies) at body temperature. > C. immitis is endemic to the southwestern US (i.e., southern and central California, Arizona, New Mexico, and western Texas), northern Mexico, and some regions of Central and South America. > C. immitis is transmitted by spore inhalation. Spores are formed by fragmentation of hyphae. Once inside the lung, the spores turn into spherules that contain endospores. The spherules subsequently rupture and release endospores that disseminate to other organs and tissues. Each endospore is capable of forming a new spherule.

Hodgkin lymphoma typically presents with lymphadenopathy and B symptoms (eg, fever, night sweats, weight loss), and histopathology classically shows _____________________________ surrounded by inflammatory cells.

Reed-Sternberg cells

uWORLD: A 23 yo man has 2 days of fever, cough, sore throat, and runny nose. His temperature is 100.4 F. Lung sounds are clear to auscultation. A nasopharyngeal swab is obtained. Naked viral particles are seen, and purified RNA molecules are extracted from these particles. Once introduced into human cells, the purified RNA molecules indue viral protein synthesis and viral genome replication. Which of the following is most likely cause of this patient's symptoms? (HIV, Influenza, RSV, rhinovirus, rotavirus).

Rhinovirus > For a purified RNA molecule to induce viral protein synthesis in a host cell, it must be able to act directly as mRNA using the host's intracellular machinery for translation. Therefore, in general, puffed single-stranded positive-sense RNA can be infectious; single-stranded negative sense or double-stranded RNA is not. > The question describes a naked (i.e., noneveloped) virus with SS(+) RNA. Of the viruses listed, only rhinovirus (a picornavirus) has these features. NOTE > Influenza A is an orthomyxoivrus that contains SS(-) RNA; similarly, respiratory syncytial virus is a paramyxovirus that contains SS(-) RNA. For these viruses to replicate in a host cell, an RNA-dependent RNA polymerase bus also gain entry into the host cell.

___________________ infection occurs when patients who have been primarily infected incompletely eliminate the M. tuberculosis bacilli within Ghon foci or extra pulmonary sites. The organism that remain then mobilize form these foci during periods of immunosuppression to cause active disease. Patients who take chronic corticosteroids, anti-TNF agents (eg. infliximab or etanercept), calcineurin inhibitors (eg, cyclosporine), or who are HIV-postive are at increased risk for reactivation tuberculosis. In the lung, the typical pathologic finding associated with secondary infection is an *apical cavitary lesion*.

Secondary M. tuberculosis

uWORLD: A 19 yo woman is brought to the ED after a motor vehicle collision. The patients medical history includes celiac disease and 3 episodes pneumonia. Her blood pressure is 80.45 and pulse is 130. Physical examination reveals pallor, and ultrasound shows a splenic laceration. She receives a blood transfusion with O-negative packed RBCs. During transfusion, the patient develops facial swelling, generalized hives and SOB. which of the following is the most likely diagnosis in this patient?

Selective IgA deficiency > The patient has a history of recurrent pneumonia, a diagnosis of celiac disease, and an anaphylactic reaction during transfusion. These findings are high suggestive of *selective IgA deficiency*, the most common primary immunodeficiency. > Although most patients with IgA definecy are asthmatic, some may have recurrent *sinopulmonary (eg, sinusitis, pneumonia) and *gastrointestinal* infections (eg, Giardia) due to absence of secretory IgA. > Concomitant autoimmune disorders (eg. celiac disease) are also common. > Patients with severe IgA deficiency can form IgE antibodies directed against IgA (anti-IgA antibodies). Serum IgA levels are low or undetectable due to failure of B cells to differentiate into IgA-secreting plasma cells; serum IgG and IgM levels are normal > When transfused with *blood products* containing small amounts of IgA, these patients can develop potentially fatal *anaphylaxis*.

____________________ results form failure of T cell development and leads to a severe defect of cell-mediated and humoral immunity. This disease causes severe bacterial, viral, and fungal infections beginning in infancy.

Severe combined immune deficiency

uWORLD: A 4-moth old girl is bough to the office for a well-cild evaluation. The girl has been growing and developing normally and her parents have no concerns. When questioned about the home environment, the parents say that they both smoke cigarettes but "try to smoke outside the house" The patient's mother smoked a pack of cigarettes a day until her second trimester, when she found out she was pregnant. However, she resumed smoking shortly after delivery and says that the stress of sleep deprivation and difficulty with postpartum weight loss have made it difficult for her to quit. The parents are counseled on the risks of secondhand smoke exposure. Which of the following conditions is the infant at highest risk for developing from exposure to cigarette smoke?

Sudden infant death syndrome > *Secondhand smoke* (SHS) has numerous adverse effects on nearly organ system. The level of tobacco in the fetus of a mother who smokes is the same as that of an active smoker. Maternal tobacco use impairs fetal oxygenation, alters fetal development and response, and exposes the fetus to multiple toxins (eg. nicotine, carbon monoxide, ammonia). > One of the most dangerous effects of SHS exposure (pre- and postnatal) is the increased risk of *sudden infant death syndrome (SIDS).

uWORLD: A biochemistry researcher conducts an experiment to determine the effects of endogenous serum factors on neutrophil-induced degradation of elastin. She uses serum samples from 2 different study subjects, subject A and subject B. During the study, she exposes neutrophils to substances that result in neutrophil activation in vitro and then adds serum from one of the study subjects. The serum from subject A appears to inhibit elastin degradation by neutrophils whereas serum from subject B does not have that ability. Patients similar to subject B should b specifically warned to avoid which of the following stimuli?

Smoking > *Neutrophil elastase* is the major protease of extracellular elastin degradation. It is related by neutrophils and macrophages. The major serum inhibitor of extracellular elastase is *alpha-1 atnitrypsin (AAT). Patient B likely has AAT deficiency, a condition associated with panacinar emphysema and liver cirrhosis. > Panacinar emphysema results from the unopposed action of neutrophil elastase on alveolar walls. > *Smoking* dramatically increases the risk of panacinar emphysema in patients with AAT deficiency by inducing inflammation (increasing neutrophil and macrophage activation) and permanently inactivating AAT through oxidation of a crucial methionine residue.

uWORLD: A 74 yo previously healthy Caucasian male comes to his physician's office complaining of abrupt onset fever, headache, myalgia, malaise, cough, and throat pain. His two granddaughters missed several days of school because of similar symptoms. Examination demonstrates mild hyperemia of the throat without any exudate, and the patient is sent home on conservative management. Fiver days later, he is admitted to the hospital with progressive dyspnea, chest pain, and productive cough. Which of the following pathogens is most likely to be isolated form this patient's sputum? (Listeria, klebsiella, Staph aureus, nontuberculous mycobacteria, cytomegalovirus)

Staphylococcus aureus > The patient's presenting signs and symptoms and report of similarly ill household children are consistent with influenza infection. Individuals infected with influenza A tend to experience abrupt onset fever, headache, myalgia, and malaise; signs and symptoms gradually improve over a period of two or five days. > A subset of patients stricken by influenza go on to develop secondary bacterial pneumonia characterized by recurrent fever, dyspnea, and productive cough. The elderly are particularly at risk for this complication. > In order, the pathogen most often responsible for secondary bacterial pneumonia are Strep pneumonia, Straph Aureus, and Haemophilus influenzae.

uWORLD: A 34 yo HIV-postive man comes to the ED with sudden-onset fever, chills, productive cough, and left-sided chest pain that worsens with deep breathing. His symptoms began 3 days ago. Physical examination shows bronchial breath sounds over the left lower lung. His most recent CD4+ lymphocyte count measured 1 month ago was 800 cells/uL. Which of the following organism is most likely responsible for they patient's symptoms? (Strep. pneumoniae, mycoplasma pneumoniae, pneumocystis jiroveci, staph aureus, legionella, moraxella catarrhalis, mycobacterium tuberculosis)

Streptococcus pneumoniae > The most common cause of community-acquired pneumonia is immunocompetent hosts (which would include an HIV+ patient with normal CD4+ counts) is Streptococcus pneumoniae. NOTE: > Because he is HIV+, it might be assumed that the cause is an AIDS-related opportunistic pathogen such as Pneumocystis jirvoveci. HOWEVER, the normal CD+ lymphocyte count for adults ranges from 400-1400 cells/uL; *counts <200 cells/uL are associated with a significantly increased risk of P jiroveci infection.*

uWORLD: A 69 yo man comes to the office due to progressive pain beneath his right scapula and in his right arm for the last several months. He has taken over-the-counter analgesics, which provide short-term relief, but the pain has become more severe and awakens him at night. The pain is associated with numbness in his right forearm extending up to the tips of the fourth and fifth fingers. The patient has also had persistent nonproductive cough and lately has been coughing up streaks of blood. He has a history of HTN, GERD, and osteoarthritis. He has smoked 2 parks of cigarettes daily for more than 40 years but recently cut down to 4 or 5 cigarettes a day. Which of the following is the most likely cause of this patient's symptoms?

Superior sulcus tumor > Tumors located in the lung apex (superior sulcus) are called Pancoast tumors. invasion of surrounding structures can lead to ipsilateral Horner syndrome, rib destruction, atrophy of hand muscles, and pain in the distribution of C8, T1, and T2 nerve roots. > This patient with an extensive smoking history who now has hemoptysis and shoulder pain likely has lung cancer, which can be associated with symptoms due to local or distant spread. *Pancoast syndrome* is caused by a tumor at the lung apex. Such tumors often arise in the *superior sulcus* (groove formed by the subclavian vessels) The apical location allows for extensive local tumor spread. Invasion of surrounding structures can cause the following: - Shoulder pain radiating toward the axilla and scapula. It occurs due to involvement of the lower brachial plexus. - Horner syndrome - involvement of the cervical sympathetic ganglia. - Spinal cord compression

uWORLD: A 3 yo male experiences recurrent sinusitis and one episode of severe pneumonia. As a part of his evaluation, Candida extract is injected intradermally. 48 hours later he returns with a firm nodule measuring 16 mm in diameter in the location where the extract was injected. Which of the following cell types is responsible for the response observed in this patient?

T-lymophcytes > Contact dermatitis, granulomatous inflammation, the tuberculin skin test and the Candidia extract skin reaction are all examples of delayed-type hypersensitivity reactions (DTH). > The cells that mediate DTH reactions are TH1-lymphocytes that realize interferon-g to cause recruitment and stimulation of macrophages . > DTH reactions take days to reach their peak activity; this is in contrast to other hypersensitivity reaction which causes clinical effects within minutes of antigen exposure.

Granuloma formation is a manifestation of cell-mediate immunity driven by productions of ____________________, particularly IL-2 and interferon-y, which stimulates TH1 type cell proliferation and macrophage activation, respectively.

TH1 type CD4+ helper T cells.

uWORLD: A 3 yo boy is brought to eh office due to a 1-day history of fever and irritability. The mother states that the boy has been tugging at his right ear. The patient has bad several episodes of otitis media in the past. Temperature is 38.1 (100.6 F). Otoscopic examination shows a bulging right tympanic membrane with erythema and exudate. Cultures from the exudate yield small, oxidase-costive, gram-negative rods that grow on factor X and V supplemented media, consistent with Haemophilus influenzae. The patient's immunization are up to date. What best explains H influenzae infection despite appropriate immunizations in this patient?

The strains responsible for this patient's disease do not produce a capsule > Nontypable strains of H. influenzae are strains that do not form an antiphagocytic capsule. They are part of the normal upper respiratory tract flora but can cause otitis media, sinusitis, and bronchitis. Immunity to nontaxable strains, as well as capsular strains other than type b, is not conferred by vaccination with the H influenzae type b vaccine.

________________ is one of the products of the cyclooxyrgenase pathway. Synthesized by platelets, it causes vasoconstriction and platelet aggregation.

Thromboxane A2

uWORLD: An 81 yo man comes to the office after an upper respiratory tract infection. His symptoms have largely resolved except for a mild nonproductive cough that is especially bothersome at night. Medical history includes HTN and knee osteoarthritis. The patient is still physically active and plays golf on the weekends. He is a retired insurance executive and has no known environmental exposures. He has no history of smoking. Physical examination is unremarkable, and chest x-ray is normal. Pulmonary function testing is considered in this patient. Compared to midlife, which of the following changes are most likely expect as a result of normal aging in this patient? TLC? FVC? RV?

Total lung capacity: Unchanged Forced Vital capacity: Decrease Residual volume: Increase > *Aging* is associated with a number of changes in pulmonary function. Patients age > 35 experience steady decrease in chest wall compliance as a result of stiffening from rib calculation and from increased thoracic curvature due to osteoporosis and osteoarthritis. > In constrast, lung compliance increases with age due to loss of elastic recoil, particularly in the alveolar ducts. > This results in a marked increase in residual volume, a decrease in forced vital capacity, and relatively unchanged total lung capacity.

uWORLD: A 56 yo Caucasian male presents to the ED with a six hour history of burning substernal pain. His past medical history is unremarkable and he takes no medications. He smokes two packs of cigarettes every day and consumes alcohol occasionally. An ECG performed in the ED reveals ST segment elevation in lead I and V3-V6. During the next several hours, the patient develops progressive shortness of breath. He is unable to lie still in the bed and insisting on sitting up. Which of the following histologic features is most likely to be newly present in this patient's lung tissue?

Transudate accumulating in the alveolar lumen > Prolonged, burning substernal pain and ST segment elevations in leads I and V3-V6 is strongly suggestive of anterolateral left ventricle infarction. Common consequences of left ventricle infraction include left ventricular failure, cariogenic acute pulmonary edema, pulmonary venous hypertension (congestion), and transudate of plasma into the lung interstitum and alveoli.

uWORLD: A 33 yo man comes to the hsotpail due to 3 days of SOB, profound fatigue, and chills. The patient has a history of IV drug abuse. He was enrolled in a methadone program but admits to relapsing last week. His temrpature is 104 F, blood pressure is 75/40, pulse is 122 and regular, and respirations are 40. He is admitted to the intensive care unit but dies despite treatment efforts. The patient's lung show infarct. What is the most likely diagnosis in this patient?

Tricuspid valve endocarditis > The lung autospy reveals multiple *wedge-shaped hemorrhagic* lesions in the periphery of the lung, which are most likely due to *pulmonary emboli*. Because there is dual blood supply to the lungs (eg, pulmonary and bronchial arteries) patients with pulmonary emboli typically develop hemorrhagic infarction as opposed to ischemic infarction > This young patient with a recent history of *intravenous drug use* and acute presentation with septic shock (eg, fever, tachycardia, tachypnea, hypotension) most likely developed *septic* pulmonary emboli from *tricuspid valve endocarditis*. Intravenous drug users are at increased risk for bacterial endocarditis involving the right-sided heart valves. Staph aureus is the most common pathogen responsible for infective endocarditis in these patients.

_________________ presents with malaise, weight loss, low-grade fever and cough. Chest X-ray shows *patchy* or *nodular infiltrates*, cavities, or calf iced nodules. Histology reveals caseating granulomas

Tuberculosis

uWORLD A 52 yo Asian male presents to your office with cough, night sweats and occasional hemoptysis. Sputum cultures placed on a selective medium grow mycobacteria microscopically observe to grow in apraelle chains ("serpentine cords") This observed bacterial growth pattern most strongly correlates with (Acid-fastness, growth rates, virulence, pigmentation, survival in macrophages)

Virulence > Cord factor is a mycoside, meaning it is composed of two mycolic acid molecules bound to the disaccharide trehalose. > The presence of cord factor correlates with virulence; mycobacteria that do not possess cord factor are not able to cause disease. > Mores specifically, cord factor is responsible for inactivating neutrophils, damaging mitochondria, and induing release of tumor necrosis factor.

__________________ is an immunodefiencecy syndrome caused by insufficient production of mature B cells, which predisposes mainly to recurrent infections with encapsulated pyogenic bacteria (eg. Streptococcu pneumoniae, Homophiles influenzae type B). The quantity and functioning of T cells is generally no affects,d so an absent timid show is unlikely.

X linking(Bruton's) agammaglobulinemia

______________ and _______________ are leukotriene D4 receptor antagonists. These agents are typically used for chronic asthma prophylaxis.

Zafirlukast ; montelukast

The presence of numerous neutrophils in the alveolar fluid would be consistent with an *exudate*, which occurs in the setting of an _______________ or _______________

acute bacterial or aspiration pneumonia

Cells of ___________________, which show *glandular* or *papillary elements* have short and plump microvilli that distinguish adenocarcinoma from mesothelioma.

adenocarcinoma

uWOLRD: A 45 yo man omes to the ED with a 1 day history of severe dyspnea on exertion. The dyspnea has been worsening throughout the day and has been occurring with progressively shorter walking distances. The patient reports no inciting trauma. He has no other medical problems and take no medications. he has a 20 pack year smoking history but does not use alcohol or illicit drugs. His blood pressure is 110/60 and pulse is 96. Arterial blood gases drawn on room air show a PaO2 of 54 and a PaCO2 of 26. Which is likely to cause of the blood gas abnormalities seen in this patient?

alveolar hyperventilation > This patient's acute onset dyspnea, hypoxemia, and hypocapnia are consistent with alveolar hyperventilation. Arterial PaCO2 is inversely related to and considered to be the main indicator of total alveolar ventilation. Assuming a normal rate of metabolic CO2 production, *hypocapnia* implies *alveolar hyperventilation* > Alveolar hyperventilation can result from a ventilation-perfusion mismatch the causes decreased O2 and CO2 exchange (eg, pneumonia or pulmonary embolism).

*Metronidazole* has excellent coverage against ____________ but does NOT cover ________ organism.

anaerobes; aerobic. > Adequate coverage against both aerobic streptococci and anaerobes is essential of successful treatment of lung abscesses.

uWORLD: A 65 yo man comes to the ED due to an episode of hemoptysis that occurred early this morning. The patient estimates that he coughed up about 100 mL of blood. Prior to this episode he was in his usually state of health and felt well, with no fevers, night sweats, or weight loss. He has had occasional episodes of blood-tinged sputum over the past few months. The patient's past medical history is significant for tuberculosis (TB) that was effectively treated several years ago. He had smoked a pack of cigarettes daily for 45 years but quite 4 years ago. A representative cut of the CT scan, shown in the image below, demonstrates changes consistent with prior TB infection, including an old left upper lobe cavity. Compared to prior scans, the main difference is that the cavity now appears to be filled with a round mass. Which of the following best describes this patient's condition? (Allergic, colonizing, contagious, invasive, malignant)

colonizing > *Aspergillus fumigatus* is a mold that is widely present in organic matter. It forms septet hyphae. The spores are inhaled with the air and are typically cleared by the mucus and ciliated epithelium of the respiratory tract. In individuals with suppressed immune defenses, Aspergilus causes a wide spectrum of disease. > This patient likely has hemoptysis due to an *aspergilloma (mycetoma)*, which represents Aspergillus *colonizaiton*. Aspergilomas develop in old lung cavities *from tuberculosis, emphysema, sarcoidosis)

uWORLD: A 10 yo boy with a history of mild persistent asthma is brought to he office for follow-up. His asthma symptoms are controlled with an inhaled beta-agonist and inhaled glucocorticoid therapy. There is no family history of asthma but both his parents and elder brother have short stature. The patient's mother is concerned about the use of glucocoorticodis because she read that it causes growth retardation on the internet. The boy's glucocortocid medication is stopped, and he is started on a trial of inhale cromolyn in addition to continuing his beta-agonist. Which of the following site of action of the new medication?

inhibits mast cell degranulation > Cromolyn and nedocromil are mast cell-stabilizing agents that inhabit mast cell degranulation independent of the triggering stimulus. They are less effective than inhaled glucocorticoids and are considered second-line for the treatment of allergic rhinitis and bronchial asthma. > These medications do not influence bronchial construction directly; therefore, they are typically used to prevent acute attacks, rathe than to treat acute exacerbations. Even though these drugs are less efficacious than inhaled glucocorticoids, they are very effective prophylactic agents for patients with seasonal symptoms, aspirin hypersensitivity, and exercise-induced asthma.

Mononuclear interstitial pulmonary infiltrates are found in the early stages of various ___________________ diseases

interstitial lung

uWORLD: A 2 day old boy develops abdominal dissension and refuses to breastfeed. He was born via normal spontaneous vaginal delivery at 39 weeks gestation to a primigravida mother. Examination shows significant abdominal distention with palpable intestinal loops. The rectum has no stool and rectal tone is normal. During the examination, the infant has several episodes of dark green emesis. Plain films of the abdomen shows air-fluid levels and small bowel dilation. Contrast enema fails to receive the obstruction, and the patient is taken for emergency laparotomy, which shows an inspissated, green fecal mass obstructing the distal ileum. The infant receivers from the surgery uneventfully. Which of the following will be the most likely cause of mortality in this patient?

pneumonia > This patient's abdominal distention, bilious emesis, and x-ray findings (air-fluid levels, small bowl dilation) are consistent with *bowel obstruction*. The intraoperative finding of a green *inspissated mass* (dehydrated meconium) in the *distal ileum* points to the diagnosis of *meconium ileum* as the source of obstruction. > Meconium ileus is a very specific finding for *cystic fibrosis* (CF) > Persistent, treatment-resistant infectious pneumonias, bronchiectasis, and for pulmonale account for most deaths due to CF.

uWORLD; A 65 yo man comes to the office due to several weeks f nonproductive cough. The patient also has anorexia and unintentional weight loss. His medical history includes hypothyroidism due to Hashimoto thyroiditis and a 50 pack year smoking history. Examination shows an enlarged, right supraclavicular lymph node. An imaging study shows a large mediastinal mass causing tracheal deviation. Histopathology of a lymph node after biopsy demonstrates clusters of small, ovoid cells with scant cytoplasm and a high mitotic count. Immunohistochemical staining is positive for chromogranin. The patient most lily suffers from which of the following conditions?

small cell lung carcinoma > Small cell carcinoma is strongly associated with smoking and is usually centrally located. This tumor arises form the primitive cell of the basal layer of the bronchial epithelium. Immunohistochemcial stains are frequently positive for neuroendocrine markers (eg. neuron-specific enolase, chromogranin, synaptophysin)

uWORLD: A 26 yo woman comes to the office for follow up. The patient and her husband want to have a child, and she inquires about the risk of certain genetic conditions, including cystic fibrosis (CF). The patient is from a small city with a stable Caucasian populaiotn, where the carrier frequency for CF is 1/30 Caucasian individuals. Her husband is from a nearby community, where CF carrier frequency in individuals of Asian decent is 1/100. Both the patient, who is Caucasian, and her husband, who is of Asian descent, are healthy. What is the portability that a child born to a mother form the Caucasian community and a father from the Asian community will have the disease?

1/12,000 > Cystic fibrosis (CF) is an *autosomal recessive* disease. The patient, who is Caucasian, has a 1/30 probability of carrying the mutant CFTR allele and her husband, who is of Asian descent, has a 1/100 probability oc carrying the same allele. If a parent is a *Carrier*, the probably the the child will inherit the mutate gall from the parent is 1/2. However, to develop CF, the child must independently inherit a mutant allele from each parent (1/2 X 1/2 = 1/4 probability). Therefore: P(affected child) = 1/2 *P(carrier mother) * P(carrier father) = 1/4 X 1/30 X 1/100 = 1/12,000.

A blood pH of ___________ is within normal range

7.35 - 7.45 pH < 7.35 = Acidemia pH > 7.45 = Alkalemia High altitude exposure results in hypoxemia with respiratory alkalosis. Over a course of a few days, chronic respiratory alkalosis sets in, with a corresponding decrease in the serum bicarbonate level reflecting renal compensation

uWORLD: A 65 yo male presents to your office with exeritonal dyspnea. he has had four respiratory infections over the course of the past year. For the past 30 years, he has smoked 1 1/2 packs of cigarettes a day. Physical examination reveals diffusely decreased breath sounds, increased chest anteroposterior diameter, and decreased diaphragmatic excursion. Pulmonary function testing will most likely show which of the following patterns of findings (TLC, total lung capacity; FEV1, forced expiratory volume in 1 second; FVC, forced vital capacity; RV, residual volume)?

> TLC: Increase > FEV1/FEV: Decrease > FVC: Decrease > RV: Increase COPD in a heavy smoker may consist of both emphysema and chronic bronchitis and thus may present with both progressive exertion dyspnea (characteristic of emphysema) and frequent respiratory infections (characteristic of chronic bronchitis). On pulmonary function testing, all COPD yields a decreased FEV1/FVC ratio. Emphysema also tends to increase TLC and TV. In contract, restrictive lung diseases can reduce lung volumes and increase FEV1/FVC

_____________ is an antiviral drug effective against herpes simplex virus (1 and 2) and varicella zoster virus. It is a necloside analog that is converted into acyclo-GTP in infected cells, which inhibits viral DNA polyermase.

Acyclovir

uWORLD: A 55 yo man is hospitalized with severe abdominal pain associated with nausea and vomiting. Lab studies show marked elevations of serum amylase and lipase. He has a history of heavy alcohol use. During hospitalization, his condition deteriorates and he develops severe respiratory distress. There are crackles bilaterally on physical examination and infiltrates bilaterally on chest x-ray. The patient's fails to improve with mechanical ventilation and 100% oxygen and dies 4 days later due to progressive respiratory failure. Which of the following autospy finding is most likely in this patient?

Alveolar hyaline membranes > This patient with *pancreatitis* and subsequent respiratory failure likely has *acute respiratory distress syndrome* (ARDS). Pancreatitis is a major risk factor for ARDS as it results in the release of large amounts of inflammatory cytokines and pancreatic enzymes into the circulation, which leads to infiltration of neutrophils into the pulmonary interstitium and alveolar spaces. Diffuse injury to the alveolar epithelium and pulmonary microvascular endothelium results in a leaky alveolocapillary membrane and significant pulmonary edema. > ARDS is typically characterized by progressive hypoxemia refractory to oxygen therapy and diffuse interstitial edema in the absence of cariogenic causes. During the first 1-6 days, interstitial and idntraalveolar edema, inflammation, and fibrin deposition cause the alveoli to become lined with waxy *hyaline membranes*. These membranes consist of fibrin exudate and inspissated protein-rich edema fluid mixed with the remnants of necrotic epithelial cells.

___________________ is a polyene anitfugnal similar to nystatin used for *systemic mycoses*. It is administered intravenously and never used for simple mucocutatneous infection due to many toxic side effect

Amphotericin B.

uWORLD: A 64 yo man is brought to the ED with worsening SOB. He is able to speak in short sentences only and desaturates with minimal exertion. He rapidly develops respiratory failure and is intubated. Despite appropriate treatment, he dies several days later in the intensive care unit. His medical history includes HTN and dyslipidemia. He smoked 1 pack a day for 40 years and worked 25 years as a nickel miner. Autospy is performed, and examination of his bronchi reveals thickened bronchial walls, neutrophilic infiltrates, mucus gland enlargement, and patchy squamous metaplasia of bronchial mucosa. Which of the following factors was likely the greatest contributor to this patient's biopsy finding? (Allergic, behavioral, genetic, infectious, neoplastic, occupational)

Behavioral > This patient with respiratory failure, hypoxia, and thickened bronchial walls with neutrophilic infiltrates and mucous gland enlargement likely has *chronic bronchitis*. Chronic bronchitis is characterized by chronic, productive cough with airflow limiatiaon and is part of the spectrum of chronic obstructive pulmonary disease. It is most commonly caused by *tobacco smoking*. Chronic irritation by other inhaled environmental substance, such as air pollutants and grain, cotton, or silica dusts, can be also contribute to its development. Biopsy typically shows *thickened bronchial walls*, neutrophilic and *lymphocytic infiltrates*, mucus gland enlargement with increased numbers of globe cells (*increasing mucus production), and patchy *squamous metaplasia* of the bronchial mucosa. NOTE: > Nickel is a carcinogen, and occupational exposure is associated with nasal dn lung cancer. However, neoplastic transformation itself does not contribute to the development of chronic bronchitis. Although this patient's nickel mining history is also suggestive of silica dust exposure, tobacco smoking is the most important risk factor for chronic bronchitis.

uWORLD: A 7 yo with a history of recurrent granulomatous skin infections and a prior episode of Aspergillus pneumonia undergoes a partial hepatoctomy to treat a poorly draining liver abscess. Genetic analysis reveals an inactivating mutation affecting a structural component of a neutrophil oxidase enzyme. This patient most likely has increased vulnerability to infections caused by which of the following pathogens?

Burkholderia cepacia > Chronic granulomatous disease (CGD) results from a genetic defect in NADPH oxidase. Normally, *NADPH oxidase* participates in the killing of microbes within neutrophil and macrophage phagolysosomes. Patients with CGD develop recurrent bacterial and fungal infections that are predominantly caused by 5 catalase-orgnaims: *Staphlocccous bureaus, Burkholderia cepacia, Serratia macescens, Nocardia, and Aspergillus.*

uWORLD: A 2 yo boy is being evaluated for persistent diarrhea. He seemed healthy until he was about 6 months old and has since experienced 4 episodes of otitis media and 3 episodes of pneumococcal pneumonia. He was at the 50th percentile for weight and height at 6 months but is now at the 25th percent for height and 10th percentile for weight. He is referred for upper GI endoscopy, and Giardia labblia is isolated from duodenal aspirates. Further workup shows very low serum levels of all immunoglobulin types. Flow cytometry of this patient's peripheral blood is most likely to show deficiency of cells bearing which of the following markers? (CD4, CD8, CD15, CD16, CD19)

CD19 > This patient's recurrent sinopulmonary infections, Giardia lamblia gastroenteritis, failure to thrive, and low immunoglobulin levels are suggestive of *X-linked agammaglobulinemia (XLA). In this condition, a mutation is the Bruton tyrosine kinase gene causes failure of bone marrow pre-B cells (CD19+ CD20+) to develop into mature, circulating B lymphocytes (CD19+, CD2-+, CD21+). > Patients with XLA have *low or absent B cells* in the peripheral blood and lymphoid tissues and *pan-hypogammaglobulinemia* (i.e., very low IgG, IgM, and IgA). As a result, they are at increased risk of infection with *pyogenic (encapsulated) bacteria*. Patients also have increased susceptibility to certain viral and parasitic infections, such as enteroviruses and Giardia lamblia, due to the absence of neutralizing antibodies NOTE > CD4 is a helper T-cell surface marker and CD8 is a cytotoxic T-cell surface marker. > CD15 is a cell surface marker present on granulocytes. It is also present on nearly all Reed-Sternberg cells and is therefore a cytologic maker useful in the diagnosis of Hodgkin lymphoma. > CD16 is a low-affinity Fc receptor found on the surface of NK cells, neutrophils, and macrophages.

uWORLD: A 36 yo man comes to the office due to chronic, mild dyspnea and fatigue. The patient also says that his mind seems "foggy" all the time and he is not able to think clearly. He has significant past medical history and does not take any medication. he lives a sedentary lifestyle and had and has smoked a half-pack of cigarettes a day for 5 years. Blood pressure is 140/85 mmHg, pulse is 84/min, and respirations are 18/min. His BMI is 31. Lung auscultation is unremarkable. Chest x-ray is normal. Arterial blood gas analysis shows partial pressure oxygen is 60 and partial pressure of CO2 is 54. His estimated alveolar to arterial oxygen gradient is 10 mmHg. Which explains this patient's lab finding?

Chronic hypoventilation > This patient with chronic fatigue, dyspnea, difficulty concentrating, and increased partial pressure of CO2 in the setting of obesity likely has *obesity hypoventilation syndrome (OHS). OHS results in a number of pathophysiologic changes, including increased CO2 production due to increased mass and surface area, sleep-disordered breathing, and reduced lung volumes and compliance. > This patient's normal A-a gradient (10 mmHg) is further evidence of OHS.

A 42 yo woman comes to the office with a nonproductive cough and worsening SOB with exertion. The patient has a history of primary pulmonary HTN and underwent lung transplantation 8 months ago. She states that she has not missed any doses of her transplant medications. A chest x-ray revels surgical evidence of her transplant but clear lung fields. Pulmonary function testing demonstrates a force expiratory volume in 1 second (FEV1), 67% of her best post transplant FEV1. Her forced vital capacity remains largely unchanged. A lung biopsy shows areas of total fibrotic obstruction in the terminal bronchioles. The patient's condition is most likely caused by which of the following?

Chronic transplant rejection > This patient with cough, dyspnea, obstructive findings on spirometry, and fibrotic destruction of the small airways likely has *chronic transplant rejection*. Lung transplantation may be complicated by both acute and chronic rejection. Chronic rejection is a major cause of morbidity and mortality and occurs in almost half of all patients within 5 years of transplantation. These patients commonly presents with dyspnea and a dry cough. > Chronic rejection affects the small bronchiole producing the obstructive lung disease known as *bronchiolitis obliterns*. Initially, histopathology shows lymphocytic inflammation and destruction of the epithelia of the small airways. Subsequently, fibrinopurlent exudate and granulation tissue are found in the lumen of the bronchiole, which ultimately results in fibrosis, scaring, and the progressive alliteration of small airways. NOTE: > Acute rejection usually occurs within the first 6 months. It is frequently asymptomatic and commonly discovered on the surveillance biopsies. Histopathology demonstrates perivascular and interstitial monocular cell infiltrates.

uWORLD: Fourth-year medical students are recruited for a research study assessing their ability to interpret biopsy samples obtained during routine clinical practice. As part of the study, they are given samples of normal respiratory mucosa and asked to identify the cell types present after staining with hematoxylin and eosin. The student observe that the respiratory epithelium changes in the composition as the airway continue distally from the trachea to the alveolar ducts. Which of the of the following features is last to disappear? (Cartilage, cilia, goblet cell, mucous gland, serous gland)

Cilia > Bronchi have a ciliated pesudostratfied columnar epithelium with mucin-secreting globe cells and submucosal mumcoserous glands. The airway epithelium gradually changes to ciliated simple cuboidal by the level of the terminal bronchioles. Bronchioles lack glands and cartilage, and the number of globe cells decreases distally, ending before the terminal bronchioles. Ciliated epithelium persists up to the respiratory bronchioles.

uWORLD: A 41 yo man comes to the ED complaining of fever, weigh loss, and a productive cough. He reports drinking 12-18 beers a day. Physical examination shows poor dentition. A foul-smelling sputum sample is expectorated and sent for Gram stain and culture. His chest x-ray show abscess. Which of the following antibiotics would be the most appropriate treatment for this patient's condition?

Clindamycin > Streptococcus pneumoniae is the most common cause of community-acquired pneumonia in the general patient population. However, alcoholic are significantly increased risk for *aspiration pneumonia* )infection with oral flora) because alcohol intoxication impairs the gag and cough reflex. Alcoholic with poor oral hygiene have increased numbers of oral bacteria, further increasing this risk. In addition, alcohol may impair the phagocytic and/or bactericidal action of alveolar macrophages, predisposing to infection. > Pulmonary infection in alcoholic patients very commonly include anaerobic oral flor (Bacteroides, Prevotella, Fusobacerium, and Peptrostreptococcus) admixed with aerobic bacteria. Necrotizing infections and lung abscesses may result. > Of the antibiotic choices given, clindamycin has the most activity against oral anaerobes and also covers aerobic Gram-postive organisms such s S. pneumoniae. > Similar to macorlides, clindamycin works primarily by binding to the 50s ribosomal subunit in bacteria and disrupting protein synthesis.

uWORLD: A 63 yo man comes to the ED with a 1 week history of progressively worsening SOB and cough. The SOB worsens when the patient lies flat in bed, and he needs to prop himself up with 3 pillows to sleep well at night. He has HTN and Type 2 DM and was treated 2 weeks ago for a MI with 2 stents placed in the left anterior descending artery. His BP is 120/80, pulse is 92, and respirations are 22. The patient's pulse ox shows 89% on room air. Chest auscultation reveals bilateral crackles at the lung bases and a third heart sound. Which of the following factor most likely accounts for this patient's dyspnea?

Decrease lung compliance > In left ventricular failure, fluid accumulation in the lung interstitum results in decreased compliance. > Dyspnea, bibasilar crackles, and presence of a S3 sound in a patient who had a recent Mi suggests *left heart failure*. > The presence of fluid in the pulmonary interstitum causes distortion (Swelling) of the affected tissue, increasing resistance to movement (*decreased lung compliance*). As a result, the negative pressure generated during normal inspiratory effector is not sufficient to adequately distend the lungs, resulting in poor gas exchange and SOB. Other causes of reduced compliance include pulmonary fibrous and insufficient surfactant.

_______________ is an autoimmune disorder that presents with proximal muscle weakness and skin involvement including a violaceous discoloration of the upper eyelids (*heliotrope rash*), and raised, violaceous, scaling eruption on the knuckles (*Gottron's sign*). CPK levels are typically elevated.

Dermatomyositis

uWORLD: Microbiology researchers conduct a series of experiments to determine how pathogenicity is transmitted among different strains of Streptococcus pneumoniae. In the first experiment, they injected nonvirulent strain A into the peritoneal cavity of the laboratory mice and observe no ill effect. In the second experiment, researchers subject virulent strain B to a detergent agent that kills and lyses the bacterial cells. They then inject the lysate into the peritoneal cavity of a new group of mice and again observe no ill effects. During the third experiment, they inject live strain A bacteria in combination with the killed strain B lysate, resulting in death of the mice. Which of the following genetic processes most likely accounts for the observed findings of these experiments?

Direct uptake of extracellular DNA > Certain strains of Streptococcus pneumoniae express capsular polysaccharides that inhibit phagocytosis, making it a successful pathogen. Strains lacking the capsule are not pathogenic; however, S pneumoniae is able to obtain new genetic material from the environment that is released following the death and lysis of neighboring bacterial cells. > This process, known as *transformation*, allows the bacterium to *take up exogenous DNA* fragments, integrate the DNA into its genome, and express the encoded proteins. Through this method, nonvirulent strains of S pneumoniae that do not form a capsule can acquire the genes that code for the capsule and therefore *gain virulence* > Bacteria that have the innate capacity to undergo transformation are said to be naturally competent and include Haemophilus, streptococcus, Bacillus, and Nisseria species.

uWORLD: A 38 yo man comes to the office with reports of dyspnea on exertion and decrease exercise tolerance. His medical history is unreliable, the has no history of childhood asthma or cough. He has never smoked and has no known occupational exposures. The patient has gained almost 100 lb over the past 4 years since leaving the military due to a sedentary lifestyle. Re currently weights 297 lb and BMI of 41. His respiratory rate is 22/min and use ox is 93% on room . His respiratory rate is 22/min and pusle 93% on room air at rest. Physical exmiantaion is notable for central obesity. Lungs and clear and auscultation bilaterally. Which of the following changes (FVC1) FVC, expiratory reserve volume (ERV), RV, and total lung capacity (TLC) are most likely to be seen on pulmonary function testing in this patient

FEV1: Decrease FVC: Decrease ERV: Decrease RV: Normal TLC: Decrease > This young patient presenting with respiratory symptoms and *central obesity* without a history of pulmonary pulmonary disease likely has obesity-related *restrictive lung disease*

uWORLD: A 32 yo Caucasian male is hostpialized after sustaining multiple long bone fractures in a motor vehicle accident. He now develops a shortness of breath, confusion, and petechiae to the upper chest. The patient dies despite resuscitation effectors. A section of his tissue with stain gives a dense black reaction, was used to demonstrate:

Fat > The development of respiratory distress, diffuse neurological impairment (eg. confusion), and an upper body petechial rash (Due to thrombocytopenia) within days of severe long bone fractures is characteristic of the fat embolism syndrome. The multiple fat emboli occluding the pulmonary microvasculature stain black with osmium tetroxide.

uWORLD: A 62 yo man comes to the physician with SOB. His symptoms have progressed over the last few months and have begun to limit his daily activities. He has no other medical problems and takes no medications. The patient drinks 3-4 alcoholic beverages a week and has a 50-pack-year smoking history. Physical examination shows decreased breath sounds and scattered wheezes bilaterally. His chest x-ray is shown below showing hyper inflated lungs and a flattened diaphragm. Which of the following parameters is most likely to be increased in this patient?

Functional residual capacity > Emphysema causes the destruction of inter alveolar walls and diminished lung elastic recoil, which results in air trapping enlarged distal airspaces. > This leads to an increase in *residual volume, functional residual capacity*, and total lung volume*.

Pulmonary vascular resistance (PVR) is lowest at the ________________________. Increased lung volumes increase PVR due to the longitudinal stretching of alveolar capillaries by the expanding alveoli. Decreased lung volumes also increase PVR due to decreased radial traction from adjacent tissue on the large extra-alveolar vessels

Functional residual capacity.

uWORLD: A previously healthy 43 yo man comes to the office during the early fall due to a 4 week history of cough. He initially had a "bad cold" that lasted about 10 days. The sneezing, runny nose, and muscle aches that accompanied the cold have improved, but the cough has persisted and worsened. The patient has "burst of coughing" for several minutes as he feels unable to clear the mucus. On at least 5 occasions, these attacks were so severe that he vomited afterward. The patient is a children's karate teacher. He received all of this childhood vaccinations but has not seen a physician in many years. There is no history of recent travels. Vital signs and physical examination are normal. Chest x-ray is unrevealing. Which of the following is the most likely cause of the patient's symptoms.

Gram-negative coccobacillus > Pertussis (whooping cough) is a highly contagious infection transmitted via respiratory droplets. It was predominately a childhood disease before vaccination became the norm; hover, many cases now occur in adolescents/adults whose immunity has waned. The diagnosis should be considered in any *adult* with acute trachobronchitis who has *not had vaccination boosters*. This patient may have contracted the disease during karate sessions with his student. Chest x-ray is unremarkable in most cases as there is no pulmonary consolidation. The phases of pertussis include: 1) Catarrhal phase - similar to many routine upper respiratory infection (eg. malaise, mild over, rhinorrhea) 2) Paroxysmal phase - marked by *severe coughing spells* and the classic inspiratory whoop or *post-tussive emesis* 3) Convalescent phase - during which the cough improves.

uWORLD: A 1 month old boy is brought to the office on a hot summer day due to vomiting and excessive sleepiness. he had been exclusively breastfeeding with normal feeding patterns until this afternoon, when he had 2 episodes of emesis and appeared much sleepier than usual. The infant was born at 41 weeks gestation after an unremarkable pregnancy. newborn screening was positive for a homozygous deltaF508 mutation affecting an epithelia transmembrane protein; further confirmatory testing is pending. Physical examination shows a somnolent boy with a sunken fontanelle and dry mucous membranes. Which of the following electrolyte disturbances is most likely responsible for this patient's current symptoms?

Hyponatremia > Patients with Cystic fibrosis (CF) produce eccrine sweat with higher than normal concentrations of sodium and chloride. Exposure t high temperature or excerise can lead to hyponatremia and hypochloremia due to excessive sodium chloride loss through sweat, therefore, salt supplication is recommended. > The CFTR preteen serves as a chloride channel that regulates the flow of sodium, chloride, and water across the epithelial mebrances of the airways, binary tree, intestines, sweat ducts, and pancreatic ducts > Pateints with CF are unable to reabsorb chloride and sodium in the eccrine ducts and therefore secrete *sweat with high sodium and chloride( levels. > The most likely cause of this patient's symptoms (lethargy, vomiting) is *hyponatremia due to excessive salt wasting* from this sweat.

uWOLRD: A 24 yo male present to your office complaining of paroxysmal episodes of breathlessness and wheezing. he does not recognize any triggers for these episodes and further explains that these events are no associated with exercise or with stress. He has no history of recent illness. He has no past medical history and takes no medications. His vitals are within normal limits, and lung examination in the office shows good air movement and no wheezing. Sputum microscopy shows many granule-containing cells and crystalloid masses. The sputum findings suggest the role of which of the following substances in this patient's condition? (IL-1, IL-3, IL-5, y-inteferon, TGF-B)

IL-5 > Paroxysmal breathlessness and wheezing in a young patient unrelated to ingestion of aspirin, pulmonary infection, inhaled irritants, stress, or/or and excerise should raise a strong suspicion for extrinsic allergic asthma. The granule-containing cells in the sputum are most likely eosinophils and the crystalloid bodies are most likely Charcot-Leyden crystals (contain eosinophil membrane protein). Chronic eosinophilic bronchitis in asthmatics involves bronchial wall infiltration by numerous activated eosinophils, largely in response to IL-5 released by allergen-activated TH2 cells.

uWORLD: A 6 yo Caucasian boy is brought to the office by his mother due to recurrent episodes of sinusitis.He also has a chronic cough that never seems to go away. The mother says that his previous pediatrician perform a "sweat test" to help determine the cause of his frequent infections, but results were normal. DNA testing was inconclusive. Despite these results, the new physician suggests that the patient has an inherited disorder that orders nasal transepithleila potential different measurements. In his test, an isotonic solution of sodium chloride is applied to the nasal mucosal surface, and the electoral potential overlying the mucosa is compared with that of the interstitial fluid. the result show a transepithelial potential difference that is more negative than normal. This patient's nasal mucosa is most likely to demonstrate which of the following physiologic changes?

Increase sodium absorption > Chronic cough and recurrent sinusitis in a young Caucasian patient should raise suspicion for cystic fibrosis (CF). The diagnosis of CF typically is based on elevated sweat chloride concentrations, characteristic clinal findings (*recurrent sinopulmonary infection,* pancreatic insufficiency). and/or a positive family history. > *CFTR channel functioning is reversed in sweat ducts compared with that in a respiratory and inessential gland.* CFTR reduces the salt content of sweat by reabsorption luminal chloride and stimulated ENaC to increase sodium absorption from the lumen into the cell. *CFTR mutations therefore result in the production of sweat with high chloride and sodium content.*

Five measurements of blood oxygen saturation taken in a fetus are: 67, 61, 56, 52 and 31%. The highest value is most likely recorded in which of the following vessels? (Ductus arteriosus, Superior vena cava, inferior vena cava, pulmonary trunk, descending aorta, umbilical artery)

Inferior vena cava > The most highly oxygenated blood in the fetus is carried by the umbilical vein, which empties directly into the inferior vena cava via the ductus venous. NOTE: > The umbilical arteries originate from the fetal internal lilac arteries and carry deoxygenated blood back to the placenta via the umbilical cord. > The descending aorta contains the same partially deoxygenated blood that is present in the fetal heart and pulmonary trunk. It delivers this blood to the fetal tissues and then to the umbilical arteries for deoxygenation.

uWORLD: A 44 yo perviously healthy man is evaluated due to progressive dyspnea over the past several years. The patient has no associated chest pain or palpitations. Physical examination shows a prolonged expiratory phase without wheeze or phonic. CT scan of the chest demonstrates bilateral lower lobe-predominant emphysema. Further testing reveals that the patient has a protease inhibitor deficiency, which has led to increase elastin fiber breakdown. Elastin fibers within alveolar walls normally allow the lung to stretch during active inspiration and recoil during passive expiration. Which of the following explains this property of elastin?

Interchain cross-links involving lysine > The rubber-like properties of elastin are due to extensive cross-linking between elastin monomers, which is facilitated by lysyl oxidase. Patients with alpha1-antitrypsin deficiency can develop early-onset, lower lobe-predominant emphysema due to excessive alveolar elastin degradation. > This patient's emphysema is likely due to *alpha1-antitrypsin deficiency*. Neutrophil-secreted elastase is an endogenous proteolytic enzyme that hydrolyzes elastin within alveolar walls. The liver synthesizes alpha1-antitryspin, a protein that inhibits neutrophil elastase and prevents alveolar wall degradation, particularly in the lower airways. Patients with alpha1-antritrypsin deficiency consequently develop excessive alveolar *elastin degradation*, which clinically manifests with *early-onset*, *lower lobe-predominant emphysema* NOTE > *Lysyl oxidase* (requires copper) oxidatively dominates some of the lysin residues of tropoelastin, facilitating the formation of *desmosine cross-links* between neighboring polypeptides. These cross-links account for the *rubber-like properties* of elastin.

uWORLD: A 34 yo man who recently emigrated from Southeast Asia comes to the physician due to persistent cough, night sweats, and weight loss from several weeks. He smokes a pack of cigarettes a day and consumes alcohol on weekends. Chest x-ray shows a right upper-lobe infiltrate. Despite antibiotic treatment for pneumonia, his symptoms worsen. Microscope should granulomas. Which of the following substance contribute most to the development of this patient's observed microscopic lesions?

Interferon-y > This patient's lung biopsy show *epitheloid macrophages* and *multinucleated giant cells*, which are the predominant cells found in *granulomas*. Granulomas often form after tissue macrophages encounter pathogens or substance that cannot be easily digest or removed. > In Mycobacterium tuberculosis infection, bacteria that becomes engulfed by macrophages are able to evade intracellular killing and survive and reproduce within phagolysosomes. However, the macrophages are still able to present mycobacterial antigens to naive T hyper cells. > Concomitant *macrophage IL-12* secretion induces the T helper cells to differentation into the *TH1 subtype*. > Mature TH1 cells then produce *interferon-y*, which activates macrophages, improving their ability to kill ingested mycobacteria. > In addition, activated macrophages produce TNF-alpha, which acts to recruit additional monocytes and macrophages to the area. These infiltrating cells cluster in a circular fashion around the remaining M tuberculosis organism, walling them in. For most individuals, this granulomatous response successfully limits the actor from spreading and erectly controls the infection.

uWORLD: A 45 yo woman is evaluated for progressive external dyspnea and fatigue. She also reports episodic pain and bluish discoloration of the finger and toes on cold exposure that improves with rewarming. Physical examination shows tightening of skin over the fingers. Cardiac examination reveals an accentuated second heart sound over the upper left sternal border. The abdomen is soft with mild hepatomegaly. There is bilateral lower extremity pitting edema. Pulmonary function test results are as follows: Normal FEV1/FVC

Intimal thickening of pulmonary arterioles > This patient's accentuated second heart sound indicates elevated pulmonary artery pressure (*pulmonary arterial hypertension [PAH]), which in turn has caused *cor pulmonale* and symptoms of right-sided heart failure (lower extremity edema and hepatomegaly with pulmonary edema). Her *sclerodactyly* (skin thickening and tightening over the fingers due to collagen deposition) and *Raynaud phenomenon* (cold-induced digital vasospasm) are features suggestive of CREST syndrome, which is localized variant of systemic scerlosis. > Both diffuse and limited forms of systemic sclerosis variants develop due to increase deposition of collagen in tissue. Increase production of collagen and extracellular matrix protein by fibroblasts. > All tissue can be affected, but the earliest damage is seen in small arterioles and capillaries. Microvascular injury of pulmonary arterioles lead to narrowing of the lumen and increased pressure in pulmonary circulation. It leads to hypertrophy of the right ventricle, with subsequent development of right-sided congestive heart failure. ANSWER/OBJECTIVE > Pulmonary hypersion develops in patients with *systematic sclerosis* as a result of damage to the pulmonary arterioles. It manifests with an accentuated pulmonary component of the second heart sound and signs of right-sided heart failure due to for pulmonale.

uWORLD: A pharmaceutical researcher is studying effects of a new drug being developed for treatment of asthma that works by reversing irritant-induced bronchoconstriciton. Laboratory guinea pigs are divided in 2 drops: a postvagotomy (complete transection of the vagus nerve) group and a normal study group. Then both groups are exposed to the irritants without receiving the study medication, and changes in the bronchial diameter and recorded. Those with the postvagotmy with the drug show decrease response. This study drug is most likely similar to which of the following medications? (Albuterol, flunisolide, ipratropium, nifedipine, theophylline, zileuton)

Ipratropium > The data impose the the medication works at the level of the parasympathetic nervous system. When the vagus nerve is stimulated, acetylcholine is realized. In airways, acetylcholine produces bronchonstriciton by acting on muscarinic receptors. Ipratropium, an anticholinergic agent and derivate of atropine, blocks the action of acetylcholine at muscarinic receptors, preventing this effect. Ipratropium, an anticholinergic agent and derivative of atropine, blocks the action of acetylcholine at muscarinic receptors, preventing bronchoconstriciton and reducing the parasympathetic situation of trachobronchial submuoscal glands in the lung.s

uWORLD: A 63 yo man is brought to ED after recent onset of high fever, confusion, headache, watery diarrhea, and mildly productive cough. He has been smoking two packs of cigarettes daily for more than 30 years and has been diagnosed with chronic bronchitis. Otherwise, his past medical history is not significant. His temperature is 40.1 C (104 F), blood pressure is 100/70, pulse is 91/min, and respirations are 28/min. Sputum Gram staining reveals numerous neutrophils but no bacteria. Which of the following is the mostly likely cause of this patient's disease?

Legionella pneumophila > Legionella pneumophilia causes Legionnaries' disease. Legionnaires' disease has a propensity to affect smokers and is characterized by very high fever, diarrhea, headache, and confusion. Laboratory studies frequently show hyponatremia. L. pneumophilia is a gram-negative rod that is often not detected on Gram stain. > Diagnosis can be difficult because the signs and symptoms are not specific. Legionella should be suspected in a patient with radiographic evidence of pneumonia, high fever, and accompany GI symptoms such as diarrhea. Acquiring a diagnostic sputum sample is difficult and unreliable., often showing few or no bacteria since unique LPS chains on the outer membrane inhbit Gram staining. > The diagnosis is most commonly made by testing for Legionella antigen in the urine. > Legionnaries's disease can cause a life threatening pneumonia if not recognized and treated properly. Treatment is the respiratory fluroquinolones (i.e. levofloxacin) or newer macrocodes (ie. azithromycin)

uWORLD: An 8 yo body with recurrent episodes of SOB and wheezing is brought to the physican's office fore evaluation. Physical examination revels middle prolong expirations. A complete blood count shows eosinophilia. The physician prescribes a pharmacologic receptor antagonist, and the symptoms improve substantially. The drug most likely acts at the receptor for:

Leukotrieine antagonist > Although numerous substances are thought to play a role in the pathogenesis of allergic asthma, only leukotrienes (LTC4, LTD4, and LTE4) and acetylcholine have pharmacologic receptor antagonists that offer clear therapeutic benefit. > Leukotrieines are synthesized by mast cells, eosinophils, basophils, and other cell types that infiltrate the bronchial mucosa in asthmatic. In addition to causing bronchial construction and hyperactivity, leukotrieines also promote mucosal edema and mucus hyper secretion. > Two leukotriene D4 receptor antagonists, zafirlukast and montelukast, may offer long-term control of atopic asthma by increasing airway caliber and reducing mucosal inflammation.

uWORLD: An 8 yo male is brough tot his pediatrician office by his mother. The child has a runny nose, sore throat, cough, and low-grade fever for the past 24 hours. The patient's mother recalls that several of the child's friends have been ill recently with similar symptoms. The most asks whether the child will need antibiotics for his condition. His pediatrician recommends symptomatic therapy and feels that his illness is most likely of viral etiology. Cytotoxic CD8+ lymphocytes are able to kill virus-infected nasal epithelial cells once sensitized. Cytotoxic CD8+ lymphocyte receptors recognize foreign proteins on the epithelial cell surface. Foreign proteins are presented on the epithelial cell surface by MHC molecules. These MHC molecules comprise which of the following components?

MHC class heavy chain and beta2-microglobulin > CD8+ cells recognize foreign antigens presented with MHC class I proteins. Each MHC class I molecule consists of a heavy chain and a beta2-microglobulin.

_________________ released by eosinophils normally function to kills helminths. It is also thought to contribute to the bronchial epithelial damage sustained by patients with atopic (extrinsic allergic) asthma

Major basic protein > Eosinophils are *bilobed nuclei* and are packed with large eosinophilic granules of relatively uniform size.

uWORLD: A 14 yo female present to your office complaining of intermittent SOB. She is unable to link the episodes to specific events or exposures. Physical examination is normal and her FEV1/FVC ration is 83%. Which of the following would be most useful in excluding asthma in this patient? (Normal chest X-ray, Negative methacholine challenge, absence of eosinophil, normal serum IgE, negative skin test to various allergens)

Negative methacholine challenge > Airway challenge testing with methacholine is a highly sensitive but nonspecific measure that can detect the degree of chroncial hyperactivity in patients suspected of having asthma. A negative methacholine challenge test can help to exclude (rule out) the diagnosis. Chest x-ray, blood eosinophil count, serum IgE level, and skin reactivity to various allergens are less sensitive; normal findings on these tests cannot exclude the diagnosis.

uWORLD: A 64 yo man comes to the office due to 4 weeks of progressive dyspnea. for the past several months, he has had a nonproductive cough and felt fatigue. His medical problems include degenerative joint disease and peptic ulcer disease. He smoked 2 packs of cigarettes daily for 38 years but quite 4 years ago. On examination, there are decreased breath sounds and percussive dullness at the base of the right lung. Chest CT scan reveals a right-sided pleural effusion and diffuse nodular thickening of the pleura. On thoracentesis, blood fluid is obtained. Pleural biopsy shows proliferation of epitheloid-type cells that are joined by desmosomes, contain abundant monofilaments, and are studded with very long microvilli. Which of the following is the most likely diagnosis?

Mesothelioma > Malignant mesothelioma is a rare neoplasm that arises from the pleura or peritoneum. It is strongly associated with asbestos exposure. Hemorrhagic pleural effusions and pleural thickening are characteristic. Histopathology reveals tumor cells with numerous, long slender microvilli and abundant tonofilaments. > *Mesothelioma* is rare malignant neoplasm arising from mesothelial cells, which line body cavities (eg, pleural, peritoneal, pericardial). > *Asbestos* exposure is the primary risk factor. Individuals involved in asbestos mining and industrial applications (eg, insulation, shipbuilding) are at risk of mesothelioma. Symptoms include dyspnea and chest pain. > *Hemorrhagic* pleural effusion are frequently present. > Histopathology will show tumor cells with numerous, *long slender microvilli* and abundant *tonofilaments*. Immunohistochemical markers (eg, pancytokeratin) are useful in diagnosis.

uWORLD: A 43 yo man comes to the office reporting occasional nighttime dyspnea that awakens him. His symptoms started approximately a year ago after a severe upper respiratory tract infection, and he now has noticed a new episodic cough. The patient has no other medical problems and no history of allergies, but his older brother has atopic dermatitis. Vital signs are normal. Physical examinations show no abnormalities. Pulmonary function tests are as follows: FEV1: Normal FEV1/FVC: Normal Diffusion capacity for CO: Normal Administration of which of the following medications should be most helpful during the remainder of this patient's diagnostic workup? (Ipratropium bromide, levalbuterol, methacholine, phenoxybenzamine, scopolamine)

Metacholine > The patient with intermittent nocturnal dyspnea, episodic cough, and family history of atopy may have *asthma* despite his normal spirometry findings. Asthma is an obstructive airway disease that occurs due to hypersensitivity of the airways to various timely, including physical, chemical, and allergenic irritants. Personal or family history of other diseases in the "allergic triad" (allergic rhinitis, atopic dermatitis, allergic asthma) can aid in raising the suspicion for asthma. The diagnosis of asthma is typically confirmed by spirometry. patients with asthma will demonstrate a decreased forced expiratory volume in 1 second (FEV1) and peak expiratory flow rate on spirometry. These changes are typically reversible with the use of a bronchodilator, usually an inhaled beta-adrenergic agonist such as albuterol. > When a patient presents with history consistent with asthma but has *normal spirometry* values, *bronchoprovocation* techniques such as *methacholine* or histamine administration, exercise, or cold air inhalation can be considered to elicit asthma symptoms and confirm the diagnosis. *Methachoine is a muscarinic cholinergic agonist that causes bronchoconstriction and increased airway secretions; a decrease in FEV1 after a methacholine challenge indicates bronchial asthma.*

In ___________________ constitutional symptoms (eg. night sweats, weight loss, fever) and chest x-ray finding (eg. upper lobe infiltrates with cavitation)

Mycobacterium tuberculosis

The ____________, ___________ and ________ vaccines are examples of capsular polysaccharide vaccines

Neisseria meningitides; Haemophilus influenzae; Streptococcus pneumonia.

uWORLD: A 63 yo woman comes to the office due to slowly progressive dyspnea on exertion and cough productive of white-yellow sputum over the past several years. She has a history of rheumatoid arthritis and seasonal allergies. The patient has smoked a pack of cigarettes daily for 40 years. BP is 135/80 and pulse is 76. Lung examination demonstrate mild bilateral expiratory wheezes, and chest x-ray reveals hyper inflated lungs. Pulmonary function test results are as following Decrease FEV1 Decrease FEV1/FVC Decrease Diffusion capacity (DLCO) Which of the following cells is most likely to play a major role in the pathogenesis of this patient's lung condition? (B lymphocytes, CD4+ lymphocytes, Eosinophils, Mast cells, Neutrophils)

Neutrophil > This patient presenting with dyspnea, productive cough, and an obstructive pattern on pulmonary function eating in the setting of a prolonged smoking history likely has *chronic obstructive pulmonary disease (COPD). COPD is characterized by chronic airway inflammation, which results in both parenchymal destruction (emphysema) and remodeling of the airways (chronic bronchitis). The primary cell lines that are increased in stable COPD are *neutrophils, macrophages, and CD8+ T lymphocytes*. These cells release enzymes and proteases such as neutrophil elastase that cause alveolar damage, reduced ciliary motion, and increased mucus secretion by goblet cells. In addition, the inflammatory cells show impaired ability to phagocytize bacterial pathogens, possibly contributing to increased risk of respiratory infections such as community acquired pneumonia > Neutrophils, macrophages, and CD8+ T lymphocytes are the primary mediators of disease in chronic obstructive pulmonary disease. They secrete enzymes and proteases that cause and perpetuate both alveolar destruction of emphysema and mucus hyper secretion found in chronic bronchitis. NOTE > Eosinophils play an important role in asthma but do not appear to have a major impact in COPD. Their increased presence in patients with cCOPD may indicate coexisting asthma.

_______________ is the drug of choice for oropharyngeal candidiasis in patients without advanced immunodeficiency. It acts acts by binding to ergosterol in the fungal cell membrane, causing the formation of pores and leakage of fungal cell contents.

Nystatin > Nystatin is not absorbed from the GI tract and is administered as an oral "swish and swallow" agent.

uWORLD: A 21 yo man with asthma comes to the office for a follow-up visit. He was recently seen at an urgent car clinic for a asthma exacerbation and was treated with nebulizer bronchodilators and a 7 day course of oral prednisone. The patient has returned to his baseline status but is currently managed with only an as-needed albuterol metered-dose inhaler. Past medical history is notable for seasonal allergies. Blood pressure is 118/76 mmHg, pulse is 64/min and respirations are 12/min. Lungs are clear to auscultation with no audible wheezes. After discussion of management options, the patient is started on a long-term, daily inhaled medication to control his asthma. Which of the following should be advised to reduced the risk of adverse drug effects in this patient?

Oral rinsing > This patient has inadequate control of asthma with short-acting beta-agonist therapy. Patients with mild intermittent asthma may be managed with as-needed bronchodilator alone. however, persistent symptoms or more severe disease also requires prophylactic use of long-term *controller medications*. *Inhaled glucocorticoids* are recommended for all such patients and have been shown to improve symptoms, reduce the need for short-acting bronchodilators, and lower the risk for serious exacerbations and associated hospitalization. > Inhaled glucocorticoids improve symptoms and reduce the risk of severe exacerbation in patients with asthma but can cause oropharyngeal candidiasis. using a spacer and rinsing the mouth after use can reduce the risk of side effects.

uWORLD: A 78 y/o nursing home resident is brought to the ED because of fever, chest pain, and a productive cough. His temperature is 39.4 (10 F), BP is 106/62 mmHg, pulse is 112/min, and respirations and 35/min. Chest x-ray reveals a left lower lobe infiltrate. Sputum microscopy shows numerous lancet-shaped gram-positive diplococci. He dies despite aggressive hydration and antibiotic treatment. This patient's death might have been prevented if he had received a vaccine containing which of the following components? (Killed bacteria, Live attenuated bacteria, Outer polysaccharide covering, recombinant surface protein, unconjugated toxoid)

Outer polysaccharide covering > The elderly and young children are at high risk for invasive pneumococcal disease and should be vaccinated against Streptococcus pneumoniae. The pneumococcal polysaccharide vaccine is an unconjugated vaccine that induces a relatively T-cell-indpendent response. In contrast, the pneumococcal conjugate vaccine contains polysaccharide material attached to a protein antigen, which allows for a more robust T-cell-dependent response.

uWORLD: Several students returning from a cave exploration trip to the Central United States develop fever, cough, and malaise. Pulmonary infiltrates and hilar adenopathy are apparent on chest x-rays. lung tissue specimens would most likely show

Ovoid cells within macrophages > Histoplasma capsulate is a dimorphic fungus that is found as mold in soil. It is also present in bird and bat droppings, and is endemic to the Mississippi and Ohio River basins. Patients may report a history of exploring caves (exposure to bats) or cleaning bird cages or coops. > H. capsulate is transmitted by the respiratory route when bird or bat droppings containing fungal spores are inhaled. In the lungs, the fungus is ingested by macrophages, and is seen on light microscopy as small intracellular oval bodies. The immune reaction is Histoplasma closely resembles that included by M. tuberculosis: a cellular response with formation of granulomata. Because the fungus largest histiocytes and the reticuloendothelial system, it may cause lymphadenopathy and hepatosplenomagly. > While the majority of immunocompetent hosts remain asymptomatic, some may develop acute pulmonary dais (cough, fever, pleurisy chest pain and pulmonary infiltrates). Furthermore, individuals with underling lung disease may develop chronic pulmonary histoplasmosis, a condition that clinically resembles tuberculosis (patients present with cough, malaise, weight loss and cavitation in the upper lung lobes). Disseminated disease occurs in immunocompromised individuals.

Alpha-1 antitrypsin deficiency causes __________________ and _________________.

Panacinar emphysema; liver cirrhosis > In *emphysema*, the lung parenchyma has *increase compliance*

uWORLD: A 45 yo man comes to the office for evaluation of a persistent cough and dyspnea on exertion. He has been a coal miner for the past 15 years and has been exposed to large quantities of dust while working underground. A lung biopsy reveals fine carbon particles within the patient's respiratory bronchioles and alveolar ducts. Which of the following defense mechanisms is most directly responsible for clearing particles from this portion of the patient's respiratory tract?

Phagocytosis > This patient has *coal workers pneumoconiosis*, a form of interstitial lung fibrosis secondary to long-term inhalation of coal particles. > The pneumoconioses are diseases resulting form the inhalation of fine dust particles that reach the respiratory bronchioles and alveoli. Particles that lodge in this region are normally cleared by alveolar macrophages. High particulate burden can cause the excessive release of cytokines from macrophages, resulting in progressive pulmonary fibrosis.

uWORLD: A 62 yo man is brought to the ED due to the sudden onset of high fever and shaking chills. Twelve hours before presentation, he had experienced SOB and cough. The patient has smoked 1 pack of cigarettes daily for 40 years. His vaccination record is unknown. Exmianaiton shows dullness to percussion over the left side of the chest. Chest x-ray reveals consolidation in the left upper lobe. Gram stain of the sputum shows gram-positive diplocci. Small alpha-hemolytic colonies grow in a blood agar plate. Which of the following is the major virulence factors of the bacteria causing the disease in this patient?

Polysaccharide capsule > The primary virulence factor of Streptococcus pneumonia (Without which it cannot cause disease) is a polysaccharid capsule that inhibits phagocytosis > This patient with fever, cough, and a consolidation on chest x-ray has *pneumonia*. Typical bacterial etiologies of community-acquired pneumonia (CAP) include *Streptococcus pneumoniae,* Haemophilus influenzae, Moraxella catarrhalis, Klebsiella pneumoniae, and Staphylococcus aureus. > Atypical pneumonia (due to Mycoplasma pneumoniae, Chlamoydophila pneumonia, and Legionella) is less likely in this patient, particularly given the acute onset and severity of his symptoms as well as the lobar consolidation on chest x-ray. > S pneumoniae, the most common cause of CAP, are gram-positive diplococci that exhibits *partial (alpha) hemolysis on blood agar (green colonies)* and are bile-solute and opochin-senstive.

uWORLD: A 30 yo woman comes to the ED with acute-onset of SOB. Analysis of the patient's expiratory gases reveals the following Tracheal pO2 = 150 mmHg Alveolar pO2 = 145 mmHg Alveolar pCO2 = 5 mmHg What best explains the results of the patient's pulmonary gas analysis?

Poor alveolar perfusion > Normal tracheal pO2 is 150 mmHg and normal alveolar pO2 is 104 mmHg. The equilibration of O2 in a normal individual at rest is perfusion-limited. Situations where O2 equilibration can become diffusion-limited include disease states such as emphysema and pulmonary fibrosis, and physiologically in states of very high pulmonary blood flow, such as during exercise.

uWORLD: A 32 yo man is hospitalized with nausea, vomiting, and severe abdominal pain. he has a history of heavy alcohol use and was admitted to the hospital for acute pancreatitis a year ago. He has continued to drink alcohol since his last hospitalization and had a party last weekend, during which he consumed an entire fifth of liquor. His temperature is 101 F, blood pressure is 110/80, pulse is 104, and respirations are 20. His abdominal examination is notable for marked tenderness in the epigastric region. His serum lipase is 2392 U/L. On the second day of hospitalization, he develops SOB and progressive hypoxemia. Chest x-ray reveals new bilateral opacities. 24 hour fluid intake is 2800 mL and urine output is 1800. Which parameter is most likely to be normal in this patient? (Capillary permeability, lung compliance, pulmonary capillary wedge pressure, ventilation to perfusion matching, work of breathing)

Pulmonary capillary wedge pressure > This patient's signs and symptoms suggest *acute respiratory distress syndrome* (ARDS), for which pancreatitis is a major risk factor. > ARDS is characterized by diffuse injury to the pulmonary microvascular endothelium and alveolar epithelium, resulting in increase pulmonary capillary permeability and leaky aveolocapillary membrane. The result in noncardiogenic pulmonary edema with a *normal* (6-12 mmHg) pulmonary capillary wedge pressure* (PCWP). An elevated PCWP would be more suggestive of cardiogenic pulmonary edema (eg, decompensated left ventricular failure). Although patients receiving large volume resuscitation can develop volume overload, this patient's adequate urine output makes this less likely. >The interstitial and idntraalveolar edema, inflammation, and hyaline membrane formation that occurs as a result of ARDS cause lung compliance to decrease, the work of breathing to increase, and the oxygen diffusion capacity of the lung to decrease. More severe involvement or atelectasis of regional alveoli can cause ventilation to perfusion mismatch with decreased ventilation in areas of maintained perfusion.

uWORLD: A 54 yo man comes to the physican due to SOB that has been slowly worsening over the last 6 months. He also has a persistent nonproductive cough. The patient does not use alcohol, tobacco, or illicit drugs. His temperature 98 F, blood pressure is 132/78, pulse is 74/min, and respirations are 16/min. Physical examination reveals fine crackles bilaterally on pulmonary auscultation and drumstick shaped finger. Chest x-ray shows diffuse reticular opacities. Pulmonary function testing reveals decreased forced vital capacity, increased FEV1/FVC ratio, and expiratory flow rates that are higher than normal when corrected for lung volume. This patient's supernormal expiratory flow rates are best explained by an increase in which of the following parameters?

Radial traction on airway walls > Interstitial lung disease is associated with decreased lung volumes and increased lung elastic recoil caused by fibrotic interstitial tissue. The increased elastic recoil results in increased radial traction *outward pulling* on the airways, leading to increased expiratory flow rates when correct for the low lung volume. > This patient's clinical presentation (progressive dyspnea, fine crackles, clubbing, and diffuse reticular opacities) is consistent with *interstitial lung disease*. Most interstitial lung diseases cause progressive *pulmonary fibrosis* with thickening and stiffening of the pulmonary intersittum. This causes increased lung elastic recoil, as well as *airway widening* due to increased outward pulling (*radial traciton*) by the surrounding fibrotic tissue. > The resulting decrease in airflow resistance leads to supernormal expiratory flow rates *higher than normal when corrected for lung volume*.

uWORLD: A 63 yo woman is hospitalized because of dyspnea on exertion and generalized fatigue. She has been hostptlaized 3 times during the past year for heart failure exacerbations. Her the remedial problems include essential hypertension for 20 years, MI at the age of 58 and hypercholesterolemia. She has a 36 pack year smoking history in addition to a 10 year history of alcohol abuse. Physical examination reveals tachycardia, distended neck veins, bilateral crackles on lung auscultation, a third heart sound on cardiac auscultation, and pedal edema. The appropriate therapy is initiated. On the third day of hospitalization, her hematocrit level increased to 50%, up from 44% on admission. An arterial blood gas analysis shows an arterial O2 partial pressure (PaO2) of 70 mmHg. A 51Cr-tagged red blood cell infusion indicates normal red blood cell mass. Which of the following is the most likely cause of this patient's polycythemia?

Relative erythocytosis > Erythrocytosis is defined as a hematocrit level > 52% in men and > 48% in women. Measurement of RBC mass is necessary to distinguish absolute from relative erythrocytosis. A normal RBC mass incites plasma volume contraction as the cause of polycythemia. NOTE > Hypoxia is a common cause of secondary erthrocytosis Sa O2 < 92% appears to be the threshold for development of secondary erythrocytosis. This patient has hypoxemia above this threshold, an acute (<3 day) increase in hematocrit, and normal RBC mass; none of these features are consistent with hypoxemia-induced erythrocytosis.

uWORLD: A 35 yo woman comes to the office due to a 3 month history of progressive dyspnea on exertion, nonproductive cough, and fatigue. She has a history of seasonal allergies for which she takes over-the counter antihistamines. The patient ascribes her symptoms to smoking cigarettes and has cut down from a pack of a day to 3 or 4 cigarettes daily. Her mother has rheumatoid arthritis. Biopsy show non-caseating granuloma. This patient has which of the following conditions?

Sarcoidosis > This patient likely has sarcoidosis, a chronic multi system disorder that leads to non-caveating granuloma deposition in various organs. It typically affects *young adults*, is more common in African Americans, and affects *women* more than men. Any organ can be affected, but the *lungs* are intimately involved in most cases. > patients often present with insidious onset of respiratory symptoms (eg. cough, dyspnea, chest pain) accompanied by fatigue, fever, and weight loss. The disease may also be incidentally decide in asymptomatic patients with typical chest x-ray findings (eg. bilateral hilar lymphadenopathy, reticular opacities).

uWORLD: A 23 yo African American female is being evaluated for fatigue and a nodular rash on her bilateral lower extremities. Chest X-ray demonstrates lung nodules and hilar fullness. Transbronchial biopsy findings include large epithelia cells, occasional giant cells and no areas of necrosis. Which of the following is the cause of this patient's condition?

Sarcoidosis > This vignette describes a typical presentation of sarcoidosis. Sarcoidosis is an inflammatory disease of unknown etiology that leads to development of non-caseaating granulomas in many organs and tissues. In sarcoidosis, non-caveating grandmas consist of aggregates of epithleoid cells (activated macrophages) and multinucleate giant cells consistent with chronic granulomatous inflammation. > The typical patient with sarcodisois is a young black woman with non-specific complaints of fever, malaise and eight loss. Lung involvement is present in the majority of patients, and manifests lineally with cough, dyspnea, and chest pain. Skin involvement is varied in presentation ranging from macules to plaques to erythema nodosum (painful skin nodules). Chest X-ray is essential for diagnosis. In stage I of sarcoidosis, it classically reveals bilateral hilar lymphadenopathy. ---> Stage IV is characterized by lung fibrosis.

uWORLD: A 72 yo man comes to the office after several days of milk headache and lethargy. He has also had weakness, chronic cough, and decreased appetites. He drinks 2 or 3 beers each weekend and has a 40 pack year smoking history. Temperature is (98.6 F), bP 120/84 and pulse is 78 and respirations are 24/min. Lung examinations show mildly prolonged expiration with end-expiratory wheezes. Labs show hyponatremia. Chest x-ray reveals overinflated lung fields and 2.5 cm left hilar mass. Biopsy of the mass would most likely show?

Small cell lung cancer > Small cell lung cancer is a neuroendocrine malignancy associated with several paraneoplastic syndromes. It is the most common cause of syndrome of inappropriate antidiuretic hormone (SIADH) due to ectopic secretion of antidiuretic hormone. SIADH is characterized by *hyponatremia*, decreased serum osmolality, and urine osmolality > 100.

uWORLD: A 35 yo African American female presents to your office with arthralgia and tender, deep nodules on her legs. She has no significant past medical history. There is hepatomegaly on physical examination. Chest x-rays reveals enlarged hilar lymph nodes, and bloodwork reveals an elevated angiotensin converting enzyme (ACE) level. Skin PPD testing shows no observable induration after 48 hours. Liver biopsy in this patient would most likely demonstrate:

Scattered granulomas > This patient's chest x-ray findings and elevated serum ACE level suggest ta diagnosis of sarcoidosis. > Sarcoidosis can cause arthralgia, and skin changes occur in 30-50%. A variety of skin lesions are possible, including subcutaneous nodules (erythema nodosium), erythematous plaques, or macules that are slightly reddened and scaling. > Liver biopsy shows changes in up to 75% of cases. Scattered granulomas are the most common liver pathology finding.

A 55 yo woman comes to the physician with fever cough, and night sweats for the past 2 weeks. Chest imaging shows a nonspecific pulmonary lesion. Transthoracic needle aspiration is performed. The specimen is cultured and grows several bacterial species, including Peptostrptococcus and Fuobacterium. Which of the following predisposing factors is most likely responsible for this patient's condition? (Mitral valve prolapse, Occult malignancy, Penetrating chest trauma, Seizure disorder, Tobacco smoking, Urinary infection)

Seizure disorder > Peptostreptococcus and Fusobacerium are anaerobic bacteria that are part of the normal mouth flora. The presence of these organisms in this patient's lung lesion is highly suggestive of a developing *lung abscess*. As the abscess evolves, it typically forms a cavitary lesion with an identifiable *air-fluid level* on imaging. Symptoms are often indolent and include fever, night sweats, weight loss, and cough productive of *foul-smelling sputum* (indicative of anaerobes) > Risk factors for lung abscess include conditions that increase aspiration risk, such as *alcoholism, drug abuse, seizure disorder, stroke, and dementia*

uWORLD: A 7 month old boy is brought to the physician by his parents due to irritability and white patches in his mouth. His past medical hoister is significant for 3 epdidoes of otitis media and 2 episodes of bronchiolitis that have required hospitalization. He also has history of chronic loose stools. The child is small for his age and ill-appearing. Head and neck examination show white patches consistent with oral candidiasis but is otherwise normal. Auscultation of the lungs show expiratory wheezing. Cardiac examination is within normal limits. Labs or normal. Serum protein electrophoresis shows a very low game globulin level. Chest x-ray reveals an absent thyme shadow. Which is the most likely diagnosis?

Severe Combined immunodeficiency > Severe combined immune deficiency (SCID) is a life-threatening immunodeficiency syndrome that presents in *infancy*. It is caused by a variety of mutations in different genes that result in impaired T and B cell development and function. > This leads to compromised cell-mediated and humoral immunity with the eventual development of *severe viral and bacterial infections* as maternal immunity wanes. Other common features include mucocutaneous candidiasis, persistent diarrhea, and failure to thrive. > Laboratory studies show very low or *absent CD3+ T cells* and *hypogammaglobulinema*. Thyme hypoplasia or aplasia is another common finding in infants with SID due to

uWORLD: A 53 yo man comes to the office due to 7-8 months of SOB. He states that he feels most short of breath when chopping wood for his fireplace. Several of his coworkers have experienced similar symptoms. he has a history of HTN and type 2 DM. Pulmonary examination reveals diffuse fine crackles. Chest x-rays reveals nodular densities in both lungs that are most prominent in the apical regions. Calcifications of the hilar lymph nodes is also seen. Bronchoscopy with transbronchial biopsy of a calcified node is performed, and polarized microscopy shows birefringent particles surrounded by dense collagen fibers. This patient most likely has a history of exposure to which of the following substance?

Silica > This middle-aged adult, presenting with dyspnea on exertion, nodular densities on x-ray, calcified hilar lymph nodes, and birefringent particles on biopsy, has silicosis. *Silicosis* is one of the several pneumonicoses, a group of interstitial lung diseases caused by the inhalation of mineral dusts. It is frequently asymptomatic but can present with dyspnea on exertion and productive cough. Symptoms usually present 10-120 years after intimal exposure. Silicosis is distinguished by calcification of the rim of hilar nodes (*eggshell calcification*) and *birefringent* silica *particles* surrounded by fibrous tissue on histology.

______________________ arises from he major bronchi. Imaging will show a hilar mass. Histologically, it is characterized by *keratinization* and *intercellular bridges*. This tumor can sometimes present as a cavitary lung lesion.

Squamous cell carcinoma

uWORLD: A professor creates a model to teach respiratory physiology to her students. In her model, two spheres with elastic properties similar to alveoli are connected with a clamp positioned between them preventing airflow. The surface tension within the two elastic spheres is identical, and neither is lined with a surfactant. Which of the following options best describes what will happen when the camp is opened?

The smaller sphere will collapse > According to Laplace's law, as the radius of a sphere with constant surface tension decease, the distending pressure increases; thus, smaller sphere collapse before larger ones. Surfactant counteracts alveolar collapse by decreasing surface tension as the alveolar radius decreases.

uWORLD: A 63 yo man with a history of chronic obstructive pulmonary disease comes to the physician for a follow-up visit. He uses a tiotropium inhaler daily and his pulmonary symptoms are sufficiently controlled. During a previous visit, he was consumed on the benefits of smoking cessation. He reports that despite trying he has been unable to stop smoking due to overwhelming cravings. The physician prescribes a drug that reduces nicotine craving while decreasing the pleasurable effects of cigarettes and other tobacco products. Which of the following medications was most likely prescribed to the patient?

Varenciline > Varenicline is a *partial agonist* of nicotinic acetylcholine receptors. It can assist patients with cessation of tobacco use by reducing withdrawal cravings and attenuating the rewarding effects of nicotine.

uWORLD: A 45 yo man is involved in a motor vehicle collision and suffers a right femoral shaft fracture. He has no other significant injuries. Upon arrival at the host pail, the patient undergoes open reduction and internal fixation of the right femur. On the third postoperative day, he comes tachycardia and tachypenic and has pleuritic chest pain. His temperature is 97F, blood pressure is 120/70 and pulse is 98. PaO2 of 65. Which is the most likely cause of this patient's hypoxemia?

Ventilation/perfusion mismatch > Sudden-onset tachypnea and chest pain in a hospitalized patient should raise suspicion for *pulmonary embolism* (PE). This patient has 2 important risk factors for PE: *immobilization* (causes venous stasis) and *recent surgery* (inflammation induces a hyper coagulable state). The risk of PE can be as high as 50% after an orthopedic procedure. Thrombi most commonly originate in the deep veins of the pelvis and the lower extremities before emboli zing to the lungs. Although the risk of *fat embolism* is also increased following a long bone fracture, this patient lacks the typical skin and neurologic findings. > Thrombic occlusion of the pulmonary circulation leads to increase blood flow to the remainder of the lung, causing a *ventilation/perfusion (V/A) mismatch*. The resulting ischemic injury also causes inflammation, leading to surfactant deficiency and atelectasis in the surrounding lung regions. This leads to high volume of deoxygenated blood traversing poorly ventilated lung regions, causing right-to-left intrapulmonary shunting which often results in *hypoxemia*.

uWORLD: The pancreas of a 22-yo Caucasian male with recurrent pulmonary infections and finger clubbing shows extensive exocrine gland atrophy and fibrosis. The pancreatic ducts are lined with squamous epithelium with areas of keratinization. The finding of squamous metaplasia in the pancreatic ducts is most likely related to a deficiency of:

Vitamin A > Recurrent sinopulmonary infections and exocrine gland fibrotic atrophy in a young Caucasian are suggestive of cystic fibrosis (CF). In the pancreas, severe CF may cause total obstruction followed by complete fibrotic atrophy of the exocrine glands. The resulting pancreatic insufficiency can cause a deficiency of fat soluble vitamins. > Avitaminosis A in particular may contribute to squamous metaplasia of the epithelial lining of pancreatic exocrine ducts, which are already injured and predisposed to squamous metaplasia by inspissated mucus. Normal levels of lining of pancreatic exocrine ducts, which are already injured and predisposed to squamous metaplasia by inspissated mucus. Normal levels of vitamin A and its metabolite, retinoid acid, are required to maintain orderly differentiation of specialized epithelia, including mucus-secreting columnar epithelium. When a deficiency state exists, the epithelium undergoes squamous metaplasia to a keratinizing epithelium.

Complete collapse of a lung usually occurs following obstruction of a *mainstem bronchus* (eg. central lung tumors in chronic smokers). As the air trapped in the lung gradually gets absorbed into the blood, there is loss of lung volume due to *alveolar collapse* (i.e., atelectasis), which causes the trachea to deviate ________________ the affected side.

toward HOWEVER > Tension pneumothorax or a large pleural effusion will cause tracheal deviation*away from* the affected lung because the excess air or fluid pushes against the mediastinal structures.

Perfusion greatly increases from the apex of the lung to the base; ventilation increases slightly from the apex to the base. For this reason, the ventilation/perfusion ratio _____________ in the lung from apex to base.

decrease > While both ventilation and perfusion are increased at the base of the lung due to the effects of gravity, the perfusion increase is far greater, causing the V/Q ratio to decrease from the apex to the base.

uWORLD: A 27 yo medical student is coughed on by a patient who suffers from an active pulmonary tuberculosis infection. The student has never been exposed to M. tuberculosis before. Which of the following would most likely happen during the first week after exposure?

intracellular bacterial prolifération > Mycobacteria are phagocytksed by alveolar macrophages, and the sulfated virulence factor expressed by M. tuberculosis allows for intracellular bacterial proliferation. > Continued proliferation eventual kills the macrophage and ashes cellular lysis, allowing M. tuberculosis to be phagocytize by and infect additional macrophages. > Virulence factors expressed by the mycobacterium promote the recuriment of additional native, inactivated macrophages and also undermine the immunologic intracellular signaling between antigen presenting cells and helper T-cells. > Eventually, antigen carrying macrophages or dendritic cells migrate to the lymph nodes and induce a helper T-cell response. However, this occurs approxaitley 2-4 weeks following the intimal infection.

uWORLD: A 57 yo man is being followed by his pulmonolgist for severe emphysema. The patient has undergone trials of several different pharmaceutical therapies. His symptoms do not satisfactorily improve despite medications and extensive follow-up, rendering him functionally disable. After being convinced that the patient has attempted maximal medical therapy, the pulmonologist refers him to a surgeon for evaluation for lung transplantation. After completing workup to ensure that there are no significant comorbid conditions, the patient undergoes allograft lung transplantation. His postoperative course is complicated by mild acute rejection, which is successful treated with immunosuppressant therapy. Which of the following is most likely top be primarily damaged by chronic rejection of this patient's allograft? (Pulmonary circulation, bronchial circulation, small airways, alveolar walls, pleural membranes)

small airways > Although ll transplant recipients receive immunosuppressive therapy, rejection reaction still commonly occurs. The severity of the rejection depends on the degree of difference between the donor and recipient major histocompatibility complex antigens in addition to other variables. > Chronic rejection can occur months to years following transplantation. In lung transplantation, the immune reaction affect the *small airways*, causing *bronchiolitis obliterates syndrome*. Symptoms include dyspnea and wheezing.


Kaugnay na mga set ng pag-aaral

Psychology Of Aging Final (Chapter 8- 14)

View Set

Psych quiz 1--all the quizzes combined

View Set

PrepU Chapter 49 Neurologic Disorders

View Set

Florida Law Enforcement Academy State Exam Review 2014.07

View Set

TBUS 400 Business Policy and Strategic Management

View Set

Chapter 8: Social Media Information Systems

View Set

IXL LEVEL J/8th DD.6 Is the pronoun reflexive or intensive?

View Set

PRO1101 Ch. 3 Abdomen (Review Questions)

View Set